Are you seeking one-on-one college counseling and/or essay support? Limited spots are now available. Click here to learn more.

How to Write the AP Lang Rhetorical Analysis Essay (With Example)

November 27, 2023

how to write AP Lang rhetorical analysis essay example

Feeling intimidated by the AP Lang Rhetorical Analysis Essay? We’re here to help demystify. Whether you’re cramming for the AP Lang exam right now or planning to take the test down the road, we’ve got crucial rubric information, helpful tips, and an essay example to prepare you for the big day. This post will cover 1) What is the AP Lang Rhetorical Analysis Essay? 2) AP Lang Rhetorical Analysis Rubric 3) AP Lang Rhetorical Analysis: Sample Prompt 4) AP Lang Rhetorical Analysis Essay Example 5)AP Lang Rhetorical Analysis Essay Example: Why It Works

What is the AP Lang Rhetorical Analysis Essay?

The AP Lang Rhetorical Analysis Essay is one of three essays included in the written portion of the AP English Exam. The full AP English Exam is 3 hours and 15 minutes long, with the first 60 minutes dedicated to multiple-choice questions. Once you complete the multiple-choice section, you move on to three equally weighted essays that ask you to synthesize, analyze, and interpret texts and develop well-reasoned arguments. The three essays include:

Synthesis essay: You’ll review various pieces of evidence and then write an essay that synthesizes (aka combines and interprets) the evidence and presents a clear argument. Read our write up on How to Write the AP Lang Synthesis Essay here.

Argumentative essay: You’ll take a stance on a specific topic and argue your case.

Rhetorical essay: You’ll read a provided passage, then analyze the author’s rhetorical choices and develop an argument that explains why the author made those rhetorical choices.

AP Lang Rhetorical Analysis Rubric

The AP Lang Rhetorical Analysis Essay is graded on just 3 rubric categories: Thesis, Evidence and Commentary, and Sophistication . At a glance, the rubric categories may seem vague, but AP exam graders are actually looking for very particular things in each category. We’ll break it down with dos and don’ts for each rubric category:

Thesis (0-1 point)

There’s nothing nebulous when it comes to grading AP Lang Rhetorical Analysis Essay thesis. You either have one or you don’t. Including a thesis gets you one point closer to a high score and leaving it out means you miss out on one crucial point. So, what makes a thesis that counts?

  • Make sure your thesis argues something about the author’s rhetorical choices. Making an argument means taking a risk and offering your own interpretation of the provided text. This is an argument that someone else might disagree with.
  • A good test to see if you have a thesis that makes an argument. In your head, add the phrase “I think that…” to the beginning of your thesis. If what follows doesn’t logically flow after that phrase (aka if what follows isn’t something you and only you think), it’s likely you’re not making an argument.
  • Avoid a thesis that merely restates the prompt.
  • Avoid a thesis that summarizes the text but does not make an argument.

Evidence and Commentary (0-4 points)

This rubric category is graded on a scale of 0-4 where 4 is the highest grade. Per the AP Lang Rhetorical Analysis rubric, to get a 4, you’ll want to:

  • Include lots of specific evidence from the text. There is no set golden number of quotes to include, but you’ll want to make sure you’re incorporating more than a couple pieces of evidence that support your argument about the author’s rhetorical choices.
  • Make sure you include more than one type of evidence, too. Let’s say you’re working on your essay and have gathered examples of alliteration to include as supporting evidence. That’s just one type of rhetorical choice, and it’s hard to make a credible argument if you’re only looking at one type of evidence. To fix that issue, reread the text again looking for patterns in word choice and syntax, meaningful figurative language and imagery, literary devices, and other rhetorical choices, looking for additional types of evidence to support your argument.
  • After you include evidence, offer your own interpretation and explain how this evidence proves the point you make in your thesis.
  • Don’t summarize or speak generally about the author and the text. Everything you write must be backed up with evidence.
  • Don’t let quotes speak for themselves. After every piece of evidence you include, make sure to explain your interpretation. Also, connect the evidence to your overarching argument.

Sophistication (0-1 point)

In this case, sophistication isn’t about how many fancy vocabulary words or how many semicolons you use. According to College Board , one point can be awarded to AP Lang Rhetorical Analysis essays that “demonstrate sophistication of thought and/or a complex understanding of the rhetorical situation” in any of these three ways:

  • Explaining the significance or relevance of the writer’s rhetorical choices.
  • Explaining the purpose or function of the passage’s complexities or tensions.
  • Employing a style that is consistently vivid and persuasive.

Note that you don’t have to achieve all three to earn your sophistication point. A good way to think of this rubric category is to consider it a bonus point that you can earn for going above and beyond in depth of analysis or by writing an especially persuasive, clear, and well-structured essay. In order to earn this point, you’ll need to first do a good job with your thesis, evidence, and commentary.

  • Focus on nailing an argumentative thesis and multiple types of evidence. Getting these fundamentals of your essay right will set you up for achieving depth of analysis.
  • Explain how each piece of evidence connects to your thesis.
  • Spend a minute outlining your essay before you begin to ensure your essay flows in a clear and cohesive way.
  • Steer clear of generalizations about the author or text.
  • Don’t include arguments you can’t prove with evidence from the text.
  • Avoid complex sentences and fancy vocabulary words unless you use them often. Long, clunky sentences with imprecisely used words are hard to follow.

AP Lang Rhetorical Analysis: Sample Prompt

The sample prompt below is published online by College Board and is a real example from the 2021 AP Exam. The prompt provides background context, essay instructions, and the text you need to analyze. For sake of space, we’ve included the text as an image you can click to read. After the prompt, we provide a sample high scoring essay and then explain why this AP Lang Rhetorical Analysis essay example works.

Suggested time—40 minutes.

(This question counts as one-third of the total essay section score.)

On February 27, 2013, while in office, former president Barack Obama delivered the following address dedicating the Rosa Parks statue in the National Statuary Hall of the United States Capitol building. Rosa Parks was an African American civil rights activist who was arrested in 1955 for refusing to give up her seat on a segregated bus in Montgomery, Alabama. Read the passage carefully. Write an essay that analyzes the rhetorical choices Obama makes to convey his message.

In your response you should do the following:

  • Respond to the prompt with a thesis that analyzes the writer’s rhetorical choices.
  • Select and use evidence to support your line of reasoning.
  • Explain how the evidence supports your line of reasoning.
  • Demonstrate an understanding of the rhetorical situation.
  • Use appropriate grammar and punctuation in communicating your argument.

AP Lang Rhetorical Analysis Essay Example

In his speech delivered in 2013 at the dedication of Rosa Park’s statue, President Barack Obama acknowledges everything that Parks’ activism made possible in the United States. Telling the story of Parks’ life and achievements, Obama highlights the fact that Parks was a regular person whose actions accomplished enormous change during the civil rights era. Through the use of diction that portrays Parks as quiet and demure, long lists that emphasize the extent of her impacts, and Biblical references, Obama suggests that all of us are capable of achieving greater good, just as Parks did.

Although it might be a surprising way to start to his dedication, Obama begins his speech by telling us who Parks was not: “Rosa Parks held no elected office. She possessed no fortune” he explains in lines 1-2. Later, when he tells the story of the bus driver who threatened to have Parks arrested when she refused to get off the bus, he explains that Parks “simply replied, ‘You may do that’” (lines 22-23). Right away, he establishes that Parks was a regular person who did not hold a seat of power. Her protest on the bus was not part of a larger plan, it was a simple response. By emphasizing that Parks was not powerful, wealthy, or loud spoken, he implies that Parks’ style of activism is an everyday practice that all of us can aspire to.

AP Lang Rhetorical Analysis Essay Example (Continued)

Even though Obama portrays Parks as a demure person whose protest came “simply” and naturally, he shows the importance of her activism through long lists of ripple effects. When Parks challenged her arrest, Obama explains, Martin Luther King, Jr. stood with her and “so did thousands of Montgomery, Alabama commuters” (lines 27-28). They began a boycott that included “teachers and laborers, clergy and domestics, through rain and cold and sweltering heat, day after day, week after week, month after month, walking miles if they had to…” (lines 28-31). In this section of the speech, Obama’s sentences grow longer and he uses lists to show that Parks’ small action impacted and inspired many others to fight for change. Further, listing out how many days, weeks, and months the boycott lasted shows how Parks’ single act of protest sparked a much longer push for change.

To further illustrate Parks’ impact, Obama incorporates Biblical references that emphasize the importance of “that single moment on the bus” (lines 57-58). In lines 33-35, Obama explains that Parks and the other protestors are “driven by a solemn determination to affirm their God-given dignity” and he also compares their victory to the fall the “ancient walls of Jericho” (line 43). By of including these Biblical references, Obama suggests that Parks’ action on the bus did more than correct personal or political wrongs; it also corrected moral and spiritual wrongs. Although Parks had no political power or fortune, she was able to restore a moral balance in our world.

Toward the end of the speech, Obama states that change happens “not mainly through the exploits of the famous and the powerful, but through the countless acts of often anonymous courage and kindness” (lines 78-81). Through carefully chosen diction that portrays her as a quiet, regular person and through lists and Biblical references that highlight the huge impacts of her action, Obama illustrates exactly this point. He wants us to see that, just like Parks, the small and meek can change the world for the better.

AP Lang Rhetorical Analysis Essay Example: Why It Works

We would give the AP Lang Rhetorical Analysis essay above a score of 6 out of 6 because it fully satisfies the essay’s 3 rubric categories: Thesis, Evidence and Commentary, and Sophistication . Let’s break down what this student did:

The thesis of this essay appears in the last line of the first paragraph:

“ Through the use of diction that portrays Parks as quiet and demure, long lists that emphasize the extent of her impacts, and Biblical references, Obama suggests that all of us are capable of achieving greater good, just as Parks did .”

This student’s thesis works because they make a clear argument about Obama’s rhetorical choices. They 1) list the rhetorical choices that will be analyzed in the rest of the essay (the italicized text above) and 2) include an argument someone else might disagree with (the bolded text above).

Evidence and Commentary:

This student includes substantial evidence and commentary. Things they do right, per the AP Lang Rhetorical Analysis rubric:

  • They include lots of specific evidence from the text in the form of quotes.
  • They incorporate 3 different types of evidence (diction, long lists, Biblical references).
  • After including evidence, they offer an interpretation of what the evidence means and explain how the evidence contributes to their overarching argument (aka their thesis).

Sophistication

This essay achieves sophistication according to the AP Lang Rhetorical Analysis essay rubric in a few key ways:

  • This student provides an introduction that flows naturally into the topic their essay will discuss. Before they get to their thesis, they tell us that Obama portrays Parks as a “regular person” setting up their main argument: Obama wants all regular people to aspire to do good in the world just as Rosa Parks did.
  • They organize evidence and commentary in a clear and cohesive way. Each body paragraph focuses on just one type of evidence.
  • They explain how their evidence is significant. In the final sentence of each body paragraph, they draw a connection back to the overarching argument presented in the thesis.
  • All their evidence supports the argument presented in their thesis. There is no extraneous evidence or misleading detail.
  • They consider nuances in the text. Rather than taking the text at face value, they consider what Obama’s rhetorical choices imply and offer their own unique interpretation of those implications.
  • In their final paragraph, they come full circle, reiterate their thesis, and explain what Obama’s rhetorical choices communicate to readers.
  • Their sentences are clear and easy to read. There are no grammar errors or misused words.

AP Lang Rhetorical Analysis Essay—More Resources

Looking for more tips to help your master your AP Lang Rhetorical Analysis Essay? Brush up on 20 Rhetorical Devices High School Students Should Know and read our Tips for Improving Reading Comprehension . If you’re ready to start studying for another part of the AP English Exam, find more expert tips in our How to Write the AP Lang Synthesis blog post.

Considering what other AP classes to take? Read up on the Hardest AP Classes .

  • High School Success

' src=

Christina Wood

Christina Wood holds a BA in Literature & Writing from UC San Diego, an MFA in Creative Writing from Washington University in St. Louis, and is currently a Doctoral Candidate in English at the University of Georgia, where she teaches creative writing and first-year composition courses. Christina has published fiction and nonfiction in numerous publications, including The Paris Review , McSweeney’s , Granta , Virginia Quarterly Review , The Sewanee Review , Mississippi Review , and Puerto del Sol , among others. Her story “The Astronaut” won the 2018 Shirley Jackson Award for short fiction and received a “Distinguished Stories” mention in the 2019 Best American Short Stories anthology.

  • 2-Year Colleges
  • Application Strategies
  • Best Colleges by Major
  • Best Colleges by State
  • Big Picture
  • Career & Personality Assessment
  • College Essay
  • College Search/Knowledge
  • College Success
  • Costs & Financial Aid
  • Dental School Admissions
  • Extracurricular Activities
  • Graduate School Admissions
  • High Schools
  • Law School Admissions
  • Medical School Admissions
  • Navigating the Admissions Process
  • Online Learning
  • Private High School Spotlight
  • Summer Program Spotlight
  • Summer Programs
  • Test Prep Provider Spotlight

College Transitions Sidebar Block Image

“Innovative and invaluable…use this book as your college lifeline.”

— Lynn O'Shaughnessy

Nationally Recognized College Expert

College Planning in Your Inbox

Join our information-packed monthly newsletter.

I am a... Student Student Parent Counselor Educator Other First Name Last Name Email Address Zip Code Area of Interest Business Computer Science Engineering Fine/Performing Arts Humanities Mathematics STEM Pre-Med Psychology Social Studies/Sciences Submit

logo-type-white

AP® English Language

3 ap® english language rhetorical essay strategies.

  • The Albert Team
  • Last Updated On: March 1, 2022

3_ap_english_language rhetorical essay strategies

The AP® English Language rhetorical essay can be nightmare inducing for some AP® students, but there is no need for fear. In this exam review we will lay out helpful strategies to get you through the rhetorical essays in no time.

Rhetorical Strategy #1: Dissecting the Prompt

The first rhetorical essay strategy is to dissect the prompt. Understanding what the rhetorical essay wants from you is essential. It is important for you to read the prompt carefully for every essay, but critical reading is even more essential to the rhetorical essay. Your rhetorical prompt that you will be given for the AP® English Language exam will contain two elements. The first element is the concrete task that the prompt is asking of you, which is always to analyze the passage that follows. The second part of the prompt is a more abstract task that is not directly asked for in the prompt, but it is implied. By completely understanding both parts of the prompt, you will be able to give a complete essay that will get you to a higher score.

One example of a prompt from an AP® English Language rhetorical essay is this one from the 2008 exam . The prompt reads:

“In the following passage from The Great Influenza, an account of the 1918 flu epidemic, author John M. Barry writes about scientists and their research. Read the passage carefully. Then, in a well-written essay, analyze how Barry uses rhetorical strategies.”

Here you can see the concrete task that the examiners are asking. They want you to analyze the passage for rhetorical strategies; however, you must figure out what you are analyzing the passage for. That is the more abstract concept that you need to dissect the prompt to find. In the case of Barry’s passage you will need to analyze how he uses rhetorical strategies in order to portray scientific research. We know this, because if you look at the prompt, it specifically states what Barry did in his work, which was to write about science and research. That is your abstract task.

Once you have found your concrete task and your abstract task, a great strategy is to write it down to keep you focused throughout your essay . Using the example above this would look like the following:

Analyze how Barry uses rhetorical strategies in order to portray scientific research.

That sentence is what you must follow when writing your essay, and if you successfully keep to this task, then you will move closer to that high score.

Rhetorical Essay Strategy #2: Stick to the Format

This next rhetorical essay strategy is the key to great organization and structure that will put your test anxiety to bed. There is a simple paragraph structure for the body paragraphs of the AP® English Language rhetorical essay that will allow you to think, write, and score higher, faster. You need to begin each body paragraph with an assertion or claim. That is the point that you are trying to make clear to your audience what you will be proving. A great example of this is from the 2006 AP® English Language rhetorical essay. Below is student 2B’s opening sentence for her first body paragraph.

“The diction of the passage fully relays Hazlitt’s position about money ( student 2B ).”

You can see how the student directly asserts what he or she will be proving in this statement. The next step in constructing your body paragraph is to give one to two pieces of textual evidence. Be sure to state why these quotations relate back to your claim, otherwise they will be deemed irrelevant by the examiners. An example of this is the next sentence in student 2B’s body paragraph about diction. Here, the student brings in elements from the text to support his or her claim about Hazlitt using diction.

“’Rejected’, ‘contempt’, ‘disparaged’, ‘scrutinized’, ‘irksome’, ‘deprived’, ‘assailed’, ‘chagrin’; the endless repetition of such discouragement shows just how emphatically Hazlitt money is requisite for happy life (student 2B).”

The final part of this strategy for conquering the body paragraphs of your rhetorical essays is to end those body paragraphs with a thorough analysis. This is the aspect of the exam where you can put your way of looking at the text into your essay.

An example of this is at the end of student 2B’s body paragraph where he or she states, “The irony of the last sentences is negative, conveying the utter hopelessness of one without money. Though one may have none in life, pitiless men will continue to mock one’s circumstances even after death! (student 2B)”

This analysis of the text adds to the textual examples above and continues to bring in new logic from the student.

When this format of a body paragraph is followed, then it is extremely effective. The essay becomes clear, assertive, and easy to follow for the examiners. Follow this rhetorical essay strategy and you are even closer to getting that 5 on the exam.

Rhetorical Essay Strategy #3: LORA

As you are looking at your AP® English Language rhetorical essay prompt and passage it is important to remember the mnemonic device, LORA. LORA stands for Language, Organization, and Rhetorical Appeals. These elements will help you form your argument.

When you read through your passage you want to think about how the author is utilizing language. Is he or she using figurative language effectively? Is there imagery within the passage? Does the diction of the passage make it more rhetorically persuasive? You should not use all of these, but picking one and analyzing it clearly in one paragraph will keep you focused on how the author uses rhetoric, which is the main task of this essay.

An example of this was in the 2006 AP® English Language rhetorical essay. Student 2A begins his or her first body paragraph with, “One of Hazlitt’s most effective methods of promoting the importance of money is his strong diction (student 2A).” This student begins his or her essay with focusing on diction as how the language is used. He or she then goes on to explain why diction betters Hazlitt’s argument, which is exactly what you must do for your own rhetorical essay.

The organization of the author is the next part of your answer to the prompt. You want to look at how the author organized his or her ideas within the passage to support his or her own argument. By pointing out the organization, or structure, of the work and how it adds to the overall persuasiveness, you will bring two of the three most important elements of rhetoric together in your essay.

After organization you need to look at the rhetoric appeals. You may know them by the names logos, pathos, and ethos. It is suggested that you cover as many of these as possible; however, if time does not permit or if the passage uses one more than the other, then you should focus on one appeal.

One example of using pathos in an essay is from student 2A from the 2006 prompt. “Hazlitt plays on the audience’s heartstrings for more than enough time to convince them of the importance of having money (student 2A).” While it would have been better for the student to directly say that this is pathos, he or she does thoroughly explain the appeal to the passions, or pathos.

Key Takeaways

When taking the AP® English Language rhetoric essay you just need to remember these three rhetorical essay strategies: dissect the prompt, follow the format, and always include LORA. If you can follow them, then you are already on your way to a 5 on the AP® English Language exam .

Let’s put everything into practice. Try this AP® English Language practice question:

Rhetorical Considerations AP® English Language Practice Question

Looking for more AP® English Language practice?

Check out our other articles on AP® English Language .

You can also find thousands of practice questions on Albert.io. Albert.io lets you customize your learning experience to target practice where you need the most help. We’ll give you challenging practice questions to help you achieve mastery of AP® English Language.

Start practicing here .

Are you a teacher or administrator interested in boosting AP® English Language student outcomes?

Learn more about our school licenses here .

Interested in a school license?​

Popular posts.

AP® Physics I score calculator

AP® Score Calculators

Simulate how different MCQ and FRQ scores translate into AP® scores

examples of ap lang rhetorical essays

AP® Review Guides

The ultimate review guides for AP® subjects to help you plan and structure your prep.

examples of ap lang rhetorical essays

Core Subject Review Guides

Review the most important topics in Physics and Algebra 1 .

examples of ap lang rhetorical essays

SAT® Score Calculator

See how scores on each section impacts your overall SAT® score

examples of ap lang rhetorical essays

ACT® Score Calculator

See how scores on each section impacts your overall ACT® score

examples of ap lang rhetorical essays

Grammar Review Hub

Comprehensive review of grammar skills

examples of ap lang rhetorical essays

AP® Posters

Download updated posters summarizing the main topics and structure for each AP® exam.

AP English Language and Composition: Sample Rhetorical Analysis and Synthesis Questions

April 9, 2024.

AP English Language and Composition: Sample Rhetorical Analysis and Synthesis Questions

The Rhetorical Analysis and Synthesis Essays are two of the three essays you’ll need to write as part of the AP English Language and Composition Exam . Read on for a sample of each, as well as tips for how to answer them. 

AP English Language and Composition: Sample Rhetorical Analysis Question

Read the following passage published back in 1967 by The New York Times. Then write an essay in which you analyze how the structure of the passage and the use of language help convey the writer’s views.

Sample Question Instructions:

  • Respond to the prompt with a thesis that may establish a line of reasoning.
  • Select and use evidence to develop and support the line of reasoning.
  • Explain the relationship between the evidence and the thesis.
  • Demonstrate an understanding of the rhetorical situation.
  • Use appropriate grammar and punctuation in communicating the argument.

Americans and Western Europeans, in their sensitivity to lingering problems around them, tend to make science and progress their scapegoats. There is a belief that progress has precipitated widespread unhappiness, anxieties, and other social and emotional problems. Science is viewed as a cold mechanical discipline having nothing to do with human warmth and the human spirit. 

But to many of us from the nonscientific East, science does not have such repugnant associations. We are not afraid of it, nor are we disappointed by it. We know all too painfully that our social and emotional problems festered long before the age of technology. To us, science is warm and reassuring. It promises hope. It is helping us at long last gain some control over our persecutory environments, alleviating age-old problems—not only physical but also, and especially, problems of the spirit.

Shiraz, for example, a city in southern Iran, has long been renowned for its rose gardens and nightingales; its poets, Sadi and Hafiz; and its mystical, ascetic philosophy, Sufism. Much poetry has been written in glorification of the spiritual attributes of this oasis city. And to be sure, Shiraz is a green, picturesque town, with a quaint bazaar and refreshing gardens. But in this “romantic” city thousands of emotionally disturbed and mentally retarded men, women, and children were, until recently, kept in chains in stifling prison cells and lunatic asylums. 

Every now and again, some were dragged, screaming and pleading, to a courtyard and flogged for not behaving “normally.” But for the most part, they were made to sit against damp walls, their hands and feet locked in chains, and thus immobilized, without even a modicum of affection from their helpless families and friends, they sat for weeks and months and years—often all their lives. Pictures of these wretched men, women, and children can still be seen in this “city of poetry,” this “city with a spiritual way of life.” 

It was only recently that a wealthy young Shirazi who, against the admonitions of his family, had studied psychology at the University of Tehran and foreign universities, returned to Shiraz and after considerable struggle with city officials succeeded in opening a psychiatric clinic, the first in those regions. After still more struggle, he arranged to have the emotionally disturbed and the mentally retarded transferred from prison to their homes, to hospitals, and to his clinic, where he and his staff now attend them. 

They are fortunate. All over Asia and other backward areas, emotionally disturbed men and women are still incarcerated in these medieval dungeons called lunatic asylums. The cruel rejection and punishment are intended to teach them a lesson or help exorcise evil spirits. 

The West, still bogged down in its ridiculous romanticism, would like to believe that emotional disturbances, dope addiction, delinquency are all modern problems brought on by technological progress, and that backward societies are too spiritual and beautiful to need the ministrations of science. But while the West can perhaps afford to think this way, the people of backward lands cannot. . . . 

. . .The obstacles are awesome, the inertia too entrenched, the people’s suffering too anguished, their impatience too eruptive. Moreover, the total cultural reorganizations such as Asia and Africa are undergoing inevitably engender their own temporary dislocations and confusions. But their goals, the direction, remain constant. We are on the move, however awkwardly at first, to a saner, better world.

How to Answer the AP English Language and Composition Rhetorical Analysis Question

Go back to the original question, which asks you to analyze two features of the passage: (1) its structure, or organization, and (2) its language. The first aspect is fairly specific. As you read the passage, you need to observe what the author discusses first, second, third, and so on. Your essay should explain not only the order of ideas but the reasons the author may have chosen that order. 

The second part of the question is more general. It invites you to analyze the use of language, which may include the author’s choice of words (diction), syntax (word order), figures of speech, use of evidence (such as statistics or logical reasoning), sentence structure, rhythm, sound, tone, or just about any other characteristics of style and rhetoric you choose. 

Although the question directs you to write about two different aspects of the passage, the essay itself should be unified. That is, a good essay should not consist of, say, two disparate paragraphs, one exclusively devoted to structure and another to language. Rather, the essay should include material that shows the interrelationship of structure and language in the passage and how those elements contribute to the meaning and effect of the passage. This might be covered in a separate paragraph, or it could be woven into the overall fabric of the essay. 

Before you begin to write, read the passage at least twice: once for an overview and once as you write your analysis. You may notice early on that the opening paragraph contains generalizations about Westerners’ concepts of science and progress. Then the author contrasts the Western view of science and progress with the Eastern view. Immediately, you see that the author, by using the first-person pronoun (as in “many of us”) is speaking from the perspective of an Easterner. Consequently, his discussion of Eastern views is apt to come across as more well-informed, more authoritative, perhaps more personal. 

To support his position, the author gives an extended example—the city of Shiraz—to illustrate just how different the East is from the West. The description and vivid images of Shiraz memorably convey the idea that the “spiritual way of life” has a side to it that many Westerners don’t know about. This is the heart of the passage. The use of quotation marks around “romantic” and “city of poetry” is meant to point out the discrepancy between the idealized and real versions of Shiraz. 

Nearing the end, the author reiterates his initial contrast between West and East, with emphasis on the East. The last paragraph offers a generalized statement about conditions in Asia and Africa, reminding the reader of the contrast made at the very beginning of the passage. Tying the end to the beginning of the passage creates a sense of unity—a desirable feature in any piece of writing.

AP English Language and Composition: Sample Argument Question

The following paragraph is adapted from Mirror for Man, a book written by anthropologist Clyde Kluckhorn in the middle of the twentieth century. Read the passage carefully. Then, write an essay that examines the extent to which the author’s characterization of the United States holds true today. Use appropriate evidence to support your argument. 

Sample Question Instructions: 

  • Respond to the prompt with a thesis that may establish a line of reasoning. 
  • Select and use evidence to develop and support the line of reasoning. 
  • Explain the relationship between the evidence and the thesis. 
  • Demonstrate an understanding of the rhetorical situation. 

Technology is valued as the very basis of the capitalistic system. Possession of gadgets is esteemed as a mark of success to the extent that persons are judged not by the integrity of their characters or by the originality of their minds but by what they seem to be—so far as can be measured by their wealth or by the variety and material goods which they display. “Success” is measured by their investments, homes, and lifestyles— not by their number of mistresses as in some cultures.

How to Answer the AP English Language and Composition Argument Question

Whether you agree, disagree, or have mixed views on the content of the passage, your job is to write a convincing argument that expresses your opinion. Initially, the word argument may suggest conflict or confrontation. But rest assured that your essay need not be combative. Rather, make it a calmly-reasoned explanation of your opinion on a debatable subject. Your goal is to persuade the reader that your opinion, supported by examples, facts, and other appropriate evidence, is correct. 

If you have strong feelings about the topic, of course you should state them in your essay. But express them in calm, rational language. Be mindful that the essay should not be an emotional rant for or against the issue. 

Consider first whether you agree with Kluckhorn’s definition of “success.” Is it, as Kluckhorn asserts, measured by income and material possessions? Or do you think that a more accurate standard of success in today’s America should be determined by less tangible criteria—things such as happiness or self-respect? Or do you stand somewhere in between those two extremes? 

The actual position you take on the issue is less crucial than your ability to support it fully by drawing from your knowledge, background, experience, or observation. Regardless of your position, be sure to include more than one example. An argument that relies on a single example, however compelling, will fall flat. 

In the prompt, Kluckhorn’s notion of success seems to refer broadly to American society. Resist responding in kind. That is, a short essay shouldn’t focus on the whole of society but only on an identifiable segment—perhaps college-educated professionals or urban, blue- collar Americans. The point is that a narrowly focused essay on a limited topic will always turn out better than one that tries to cover too much ground in just a few paragraphs.

AP Biology Resources

  • About the AP Biology Exam
  • Top AP Biology Exam Strategies
  • Top 5 Study Topics and Tips for the AP Biology Exam
  • AP Biology Short Free-Response Questions
  • AP Biology Long Free-Response Questions

AP Psychology Resources

  • What’s Tested on the AP Psychology Exam?
  • Top 5 Study Tips for the AP Psychology Exam
  • AP Psychology Key Terms
  • Top AP Psychology Exam Multiple-Choice Question Tips
  • Top AP Psychology Exam Free Response Questions Tips
  • AP Psychology Sample Free Response Question

AP English Language and Composition Resources

  • What’s Tested on the AP English Language and Composition Exam?
  • Top 5 Tips for the AP English Language and Composition Exam
  • Top Reading Techniques for the AP English Language and Composition Exam
  • How to Answer the AP English Language and Composition Essay Questions 
  • AP English Language and Composition Exam Sample Essay Questions
  • AP English Language and Composition Exam Multiple-Choice Questions

AP Human Geography Resources

  • What’s Tested On the AP Human Geography Exam?
  • AP Human Geography FAQs
  • AP Human Geography Question Types and Strategies
  • Top 5 Study Tips for the AP Human Geography Exam

FOLLOW ALONG ON SOCIAL

pep

Find what you need to study

Rhetorical Devices List w/ Examples

37 min read • may 10, 2022

Sumi Vora

List of Rhetorical Devices & Terms

Taking AP English Language? This is a list of main rhetorical device terms that you should know for the exam as well as definitions & examples for each. These terms will mostly show up on the multiple-choice section, so it’s important to be able to identify them in a work of writing, but you won’t actually have to use the device in your own writing. Each term includes a definition, an example of the rhetorical device being used in a text, and an example of analysis that might be used in an essay.

In your essays, you will need to identify which devices are used and their effect on the work as a whole. Sometimes, a writer will use a device (for example: alliteration), but it doesn’t have a huge effect on the work or the writer’s argument. In that case, don't spend an entire paragraph talking about alliteration. You need to focus on what matters most, and you need to specifically show how  these choices make the work effective, and why they are so important. Yes, the rhetorical analysis essay is an argument essay just like the other two.

You aren't required to use rhetorical vocabulary in your essays at all — in fact, it’s probably better if you don’t. If you force the vocabulary into your essay, you risk sounding clunky, and the vocabulary almost always leads you to switch to passive voice. Instead, just describe what is happening! (ex: The author uses imagery → The author’s vivid images). This method also ensures that you are showing how the device is contributing to the work, rather than simply identifying it.

And, without further ado… Here are some rhetorical devices you should know for the AP Lang exam:

1. aesthetic

Definition: This rhetorical device references to artistic elements or expressions within a textual work

Example of aesthetic: 

“The Flapper” by Dorothy Parker (1922)

The Playful flapper here we see,

The fairest of the fair.

She's not what Grandma used to be, —

You might say, au contraire.Her girlish ways may make a stir,

Her manners cause a scene,

But there is no more harm in her

Than in a submarine.

She nightly knocks for many a goal

The usual dancing men.

Her speed is great, but her control

Is something else again.

All spotlights focus on her pranks.

All tongues her prowess herald.

For which she well may render thanks

To God and Scott Fitzgerald.

Her golden rule is plain enough —

Just get them young and treat them

Analysis:  Parker describes the aesthetic  of flapper culture in her poem in order to support women who defied social norms and who adopted more liberal attitudes towards makeup, drinking, smoking, and sex.

Note: aesthetic is not necessarily a specific device; it is the bigger picture. An author would use a rhetorical device (e.g. imagery, allusions, etc.) to achieve a certain aesthetic.

2. allegory

Definition: This rhetorical device references the expression by means of symbolic fictional figures and actions of truths or generalizations about human existence

Allegory Example:  

Animal Farm  by George Orwell (1945)

All that year the animals worked like slaves. But they were happy in their work; they grudged no effort or sacrifice, well aware that everything they did was for the benefit of themselves and those of their kind who would come after them, and not for a pack of idle, thieving human beings.

Analysis:  In George Orwell’s allegorical  novel Animal Farm , overworked farm animals rise up against their owner and subscribe to the concepts of Animalism, which proclaims that “all men are enemies” and “all animals are comrades.” The animals, who now work “like slaves” for the “benefit of themselves and those of that their kind,” run a society that mirrors that of the Russian Revolution. Orwell’s use of animals to describe contemporary political events creates distance between his novel and his potentially incendiary critique of the rise of Communism, which makes the topic more approachable.

3. alliteration  

Definition: This rhetorical device references the repetition of the same sound at the beginning of successive words

Alliteration Example:  

Ronald Reagan’s Address at the Vietnam Veteran’s Memorial (1988)

Our liberties, our values — all for which America stands — is safe today because brave men and women have been ready to face the fire at freedom's front. And we thank God for them.

Analysis:  Reagan acknowledges that the veterans of the Vietnam War were prepared to “face the fire at freedom’s front.” Through his use of alliteration , Reagan emphasizes the soldiers’ willingness to sacrifice themselves for freedom, focusing the audience’s attention on the value of the veterans’ deeds.

4. allusion

Definition: This rhetorical device is a reference, explicit or implicit, to something in previous literature or history

Allusion Example:  

“I Have a Dream” by Martin Luther King, Jr. (1963)

Five score years ago, a great American, in whose symbolic shadow we stand today, signed the Emancipation Proclamation. This momentous decree came as a great beacon light of hope to millions of Negro slaves who had been seared in the flames of withering injustice. It came as a joyous daybreak to end the long night of their captivity.

Analysis:  King begins his speech with both an indirect and direct allusion  to Abraham Lincoln’s “Emancipation Proclamation.” The first phrase of King’s speech, “Five score years ago,” directly mirrors Lincoln’s historic speech, which opens with “four score and seven years ago.” By associating himself with a prominent figure in the fight against injustice, King implies that he shares Lincoln’s values and establishes a sympathetic relationship with his audience.

5. ambiguity

Definition: This rhetorical device references a word, phrase, or sentence whose meaning can be interpreted in more than one way

Ambiguity Example:  

The Awakening  by Kate Chopin (1899)

Exhaustion was pressing upon and overpowering her.

"Good-by— because I love you." He did not know; he did not understand. He would never understand. Perhaps Doctor Mandelet would have understood if she had seen him — but it was too late; the shore was far behind her. And her strength was gone.

Analysis:  At the end of Kate Chopin’s The Awakening , Edna lends herself to the tide with the vague last words, “good-by— because I love you,” leaving Victor to question whether her death was intentional. Chopin’s use of ambiguity  to depict Enda’s death illustrates Victor’s lack of closure and his feeling of utter helplessness and confusion as he watches his loved one, both physically and metaphorically, swept away by the current.

Definition: This rhetorical device references an extended comparison between two things/instances/people etc. that share some similarity to make a point

Analogy Example:  

“What True Education Should Do” by Sydney J. Harris (1994)

Pupils are more like oysters than sausages. The job of teaching is not to stuff them and then seal them up, but to help them open and reveal the riches within. There are pearls in each of us, if only we knew how to cultivate them with ardor and persistence.

Analysis:  Harris compares students to oysters whom we should help “open and reveal the riches within.” Through her analogy , Harris establishes a basis on which readers can shift their perspective. Rather than simply listing specific traits of students, Harris helps her readers change their perception of how students should be treated, and gives readers a concrete and memorable lense through which readers should view the classroom.

7. anaphora

Definition: This rhetorical device references repetition of the same word or group of words at the beginning of successive clauses, sentences, or lines

Anaphora Example:  

I have a dream that one day even the state of Mississippi, a state sweltering with the heat of injustice, sweltering with the heat of oppression, will be transformed into an oasis of freedom and justice. I have a dream that my four little children will one day live in a nation where they will not be judged by the color of their skin but by the content of their character.

I have a dream today.

Analysis:  King repeats the phrase, “I have a dream” to emphasize his vision for racial equality in the United States. By employing anaphora  to underscore his beliefs, King connects his ideas with a common motif, helping his audience follow his speech and make it more memorable. King thus invites his audience to share in his “dream,” as he reminds them that it is their dreams for a more equal future that unite their movement.

8. anecdote

Definition: This rhetorical device references a usually short narrative of an interesting, amusing, or biographical incident

Anecdote Example:  

“Gender Equality is Your Issue Too” by Emma Watson (2014)

I started questioning gender-based assumptions when at eight I was confused at being called “bossy,” because I wanted to direct the plays we would put on for our parents—but the boys were not. When at 14 I started being sexualized by certain elements of the press. When at 15 my girlfriends started dropping out of their sports teams because they didn’t want to appear “muscly.” When at 18 my male friends were unable to express their feelings. I decided I was a feminist and this seemed uncomplicated to me.

Analysis:  By sharing a short anecdote  about being “sexualized” and called “bossy,” while acknowledging her male friends being “unable to express their feelings,” Watson establishes her authority to speak on gender-related issues, and she appeals to her audience’s sense of emotion and empathy as she aims to establish a common experience between both men and women in the United Nations.

9. antithesis

Definition: This rhetorical device references the rhetorical contrast of ideas by means of parallel arrangements of words, clauses, or sentences

Antithesis Example:  

Neil Armstrong’s moon landing (1969)

“That’s one small step for man; one giant leap for mankind”

Analysis:  Armstrong’s antithesis  serves to highlight the monumental impact that the moon landing will have on the human race. By contrasting his “small step” with the “giant” effect that this step will have, he emphasizes its significance.

10. assonance

Definition: the repetition of vowel sounds but not consonant sounds

Assonance Example:  

The Color Purple  by Alice Walker (1982)

She got sicker an sicker.

Finally, she ast Where it is?

I say God took it.

He took it. He took it while I was sleeping. Kilt it out there in the woods. Kill this one too, if he can.

Analysis:  In her second letter to God, Celie describes her mother getting “sicker an sicker” and the way God “kilt” her first child in the woods. The repetition of the “i” sound creates a staccato and rhythmic quality to the letter while still creating a thin, ill-sounding intonation.

Note: assonance is often associated with euphony : soothing and pleasant sounds.

11. asyndeton

Definition: conjunctions are omitted, producing a fast-paced and rapid prose

Asyndeton Example:  

“Duty, Honor, Country” by General Douglas MacArthur (1962)

Duty, Honor, Country: Those three hallowed words reverently dictate what you ought to be, what you can be, what you will be. They are your rallying points: to build courage when courage seems to fail; to regain faith when there seems to be little cause for faith; to create hope when hope becomes forlorn.

Analysis:  In his speech, MacArthur rallies the United States army with three simple words: “duty, honor, country.” MacArthur’s asyndeton  creates a powerful and concise phrase that galvanizes his men through its simplicity. Because the conjunctions have been omitted, MacArthur’s phrase reads like a chant in which each word is emphasized equally. This rhythmic phrase is thus very easy to remember and to repeat, which allows MacArthur to invigorate and prepare his army.

12. chiasmus

Definition: repetition of ideas in inverted order

Example:  John F. Kennedy’s Inaugural Address (1971)

The energy, the faith, the devotion which we bring to this endeavor will light our country and all who serve it — and the glow from that fire can truly light the world. And so, my fellow Americans: ask not what your country can do for you — ask what you can do for your country.

Analysis:  In his 1971 Inaugural Address, Kennedy encourages his audience to have faith in their generation and in their country in the midst of a trying Cold War. Kennedy attempts to unite the audience under a national identity and purpose, inviting them to consider not what their “country can do for” them, but what they “can do for” their country. By employing chiasmus , Kennedy highlights the difference between an archaic mentality and the attitude that he wants the country to adopt moving forward. Because Kennedy repeats the same simple ideas, he also creates a memorable phrase that allows his message to spread easily among the American people.

13. colloquial

Definition: characteristic of spoken or written communication that seeks to imitate informal speech

Example:  Barack Obama’s message about political ‘wokeness’ (2019)

This idea of purity and you’re never compromised and you’re always politically woke and all that stuff; you should get over that quickly. The world is messy. There are ambiguities.

Analysis:  In his commentary regarding the call-out culture on the current socio-political stage, Obama uses the term “woke” to describe those who believe they are more aware of social injustices. By adopting a colloquial  expression, Obama molds his message to resonate with young Americans. Obama is thus able to connect with his audience by mimicking their language.

14. connotation

Definition: the set of associations implied by a word in addition to its literal meaning

Example:  “Black Men in Public Space” by Brent Staples (1986)

My first victim was a white woman, well dressed, probably in her early twenties. I came upon her late one evening on a deserted street in Hyde Park, a relatively affluent neighborhood in an otherwise mean, impoverished section of Chicago. As I swung onto the avenue behind her, there seemed to be a discreet, noninflammatory distance between us. Not so. She cast back a worried glance. To her, the youngish black man – a broad six feet two inches with a beard and billowing hair, both hands shoved into the pockets of a bulky military jacket – seemed menacingly close.

Analysis:  In his essay “Black Men in Public Space,” Brent Staples refers to the woman who runs away from him as his “victim” to whom he is “menacingly close,” which connotes violence and criminal activity. However, the actions that ensue do not match such connotations ; rather than attacking the woman, Staples simply walks down the avenue. By breaking the audience’s expectations, Staples highlights the misleading dialogue surrounding African-American men and forces his readers to confront their own racial biases.

Note: connotation and tone are very closely related. Often, an author will use words that carry certain connotations to establish a tone. You can use this idea in your essays to demonstrate tone by citing the connotative words the author uses to establish such a tone.

15. consonance

Definition: the repetition of consonant sounds, but not vowels, as in assonance

Example: “Jabberwocky” by Lewis Caroll (1871)

“Beware the Jabberwock, my son!

The jaws that bite, the claws that catch!

Beware the Jubjub bird, and shun

The frumious Bandersnatch!”

Analysis:  In Lewis Carol’s poem “Jabberwocky,” he warns against the Jabberwock’s “jaws” and the “Jubjub bird,” repeating the “j” sound. Carol uses consonance  to create dissonant and almost disorienting sounds through harsh, hard tones, which emphasize the obnoxious nature of the Jabberwocky. Because of the abundance of consonants, the poem reads similar to a tongue-twister, which further serves to disorient the reader and make them feel as if they are in a completely different world.

Note: consonance can be associated with cacophony, or harsh, discordant sounds, if it uses “explosive consonants” such as B, C, CH, D, G, J, K, P, Q, T, X.

16. deductive reasoning

Definition: reasoning that works from the more general to the more specific, beginning with a theory that becomes a hypothesis, and using observations to confirm the original theory (top-down approach)

Example:  Mahatma Gandhi’s letter to British Viceroy Lord Irwin (1930)

If I have equal love for your people with mine, it will not long remain hidden. It will be acknowledged by them, even as the members of my family acknowledged after they had tried me for several years. If the people join me, as I expect they will, the sufferings they will undergo, unless the British nation sooner retraces its steps, will be enough to melt the stoniest hearts. The plan through civil disobedience will be to combat such evils as I have sampled out. If we want to sever the British connection it is because of such evils. When they are removed, the path becomes easy. Then the way to friendly negotiation will be open. If the British commerce with India is purified of greed, you will have no difficulty in recognizing our independence.

Analysis:  In his letter to Lord Irwin, Gandhi uses a series of if-then statements to defend India’s call for independence through civil disobedience. Gandhi begins by establishing his “equal love” for the British people and mentioning that if they join him in his protests, it will “melt the stoniest of hearts” in the British government, forcing the British to “retrace their steps” and remove the “evils” in the current British regime. If the evils are removed, Gandhi promises, the “way to friendly negotiation will be open.” By articulating his position with deductive reasoning , Gandhi appeals to Lord Irwin’s logic and maintains that the Indian people are not acting irrationally. Gandhi provides Lord Irwin with only one logical option: purify the British commerce system of greed and open the table to negotiate with India.

17. denotation

Definition: the literal meaning of a word, the dictionary definition

Example:  “Gender Equality is Your Issue Too” by Emma Watson (2014)

I was appointed six months ago and the more I have spoken about feminism the more I have realized that fighting for women’s rights has too often become synonymous with man-hating. If there is one thing I know for certain, it is that this has to stop.

For the record, feminism by definition is: “The belief that men and women should have equal rights and opportunities. It is the theory of the political, economic and social equality of the sexes.

Analysis:  By explicitly defining feminism as “the belief that men and women should have equal rights and opportunities,” Watson juxtaposes the denotation  of feminism with the connotations with which it is associated. Watson directly confronts the misconceptions regarding feminism to quell any opposition regarding such misconceptions, and she appeals to a credible source — the dictionary — to support her claims and establish her own authority over the matter.

Note: denotation is almost always used in contrast with connotation. Authors will often define a word to clarify its meaning, which suggests that the connotations of the term do not match how the author wants the audience to view that term.

18. diction

Definition: a writer's choice of words, phrases, sentence structures, and figurative language, which combine to help create meaning

Example:  “On Dumpster Diving” by Lars Eighner (1992)

Canned goods are among the safest foods to be found in Dumpsters but are not utterly foolproof. Although very rare with modern canning methods, botulism is a possibility. Most other forms of food poisoning seldom do lasting harm to a healthy person, but botulism is almost certainly fatal and often the first symptom is death. Except for carbonated beverages, all canned goods should contain a slight vacuum and suck air when first punctured. Bulging, rusty, and dented cans and cans that spew when punctured should be avoided, especially when the contents are not very

acidic or syrupy.

Analysis:  Eighner employs empirical diction  to describe the process of dumpster diving, which is generally considered a dishonorable and crude practice. Eighner details the “fatal” effects of “botulism,” and provides a practical assessment of “modern canning methods,” instructing readers to avoid “bulging, rusty, and dented cans” and to look for a “slight vacuum” in canned goods. By analyzing the process of dumpster diving through a scientific lens, Eighner emphasizes that those who dumpster dive are not inferior to their store going counterparts, and he suggests that dumpster diving can be a practical hobby for anyone, even if it is not done out of necessity.

19. didactic

Definition: tone; instructional, designed to teach an ethical, moral, or religious lesson

Example:  “Advice to Youth” by Mark Twain (1882)

First, then. I will say to you my young friends — and I say it beseechingly, urgently — Always obey your parents, when they are present. This is the best policy in the long run because if you don’t, they will make you. Most parents think they know better than you do, and you can generally make more by humoring that superstition than you can by acting on your own better judgment.

Analysis:  In his satire “Advice to Youth,” Twain adopts a didactic  tone that mimics that of many parents chastising their children. He instructs youth to “always obey [their] parents” because “most parents think they know better than” their children. By using a familiar instructional tone while mocking parental attitude, Twain appeals to his credibility by establishing that he too has faced criticism from his parents. By recognizing a common experience, Twain builds a rapport with his young audience, making them more receptive to his message.

Note: Generally, essays with a very didactic tone are ineffective, so they don’t have much rhetorical merit. Twain’s speech is instead a satire of the didactic tone many parents adopt, which allows him to connect with his audience in their mutual scorn for some parents’ sanctimonious attitude.

20. elegiac

Definition: a tone involving mourning or expressing sorrow for that which is irrecoverably past

Example:  Ronald Reagan’s address following the explosion of the Challenger Space Shuttle (1986)

Today is a day for mourning and remembering. Nancy and I are pained to the core by the tragedy of the shuttle Challenger. We know we share this pain with all of the people of our country. This is truly a national loss. For the families of the seven, we cannot bear, as you do, the full impact of this tragedy. But we feel the loss, and we're thinking about you so very much.

Analysis:  At the beginning of his address, Reagan adopts an elegiac  tone, declaring that “today is a day for mourning and remembering.” He describes the deaths of the astronauts as a “national loss” that pains “all of the people” in the United States. By taking the time to recognize the tragic loss of the astronauts and by empathizing with the American people’s shock at the explosion, Reagan appeals to his audience’s grief and establishes an emotional connection with them before he begins speaking about the future of the United States space exploration program.

21. epistrophe

Definition: ending a series of lines, phrases, clauses, or sentences with the same word or words

Example:  Madelynn Albright’s commencement speech for Mount Holyoke College (1997)

As you go along your own road in life, you will, if you aim high enough, also meet resistance, for as Robert Kennedy once said, “if there’s nobody in your way, it’s because you’re not going anywhere.” But no matter how tough the opposition may seem, have courage still—and persevere.

There is no doubt, if you aim high enough, that you will be confronted by those who say that your efforts to change the world or improve the lot of those around you do not mean much in the grand scheme of things. But no matter how impotent you may sometimes feel, have courage still — and persevere.

It is certain, if you aim high enough, that you will find your strongest beliefs ridiculed and challenged; principles that you cherish may be derisively dismissed by those claiming to be more practical or realistic than you. But no matter how weary you may become in persuading others to see the value in what you value, have courage still—and persevere.

Inevitably, if you aim high enough, you will be buffeted by demands of family, friends, and employment that will conspire to distract you from your course. But no matter how difficult it may be to meet the commitments you have made, have courage still—and persevere.

Analysis:  In her commencement speech, Albright encourages women to stand firm and to “aim high,” despite the prevalence of gender inequality. Albright recognizes that women face opposition and glass ceilings, but she urges them to “have courage still— and persevere,” repeating the phrase after each challenge she discusses. Like her attitude towards success, Albright’s speech always returns to the idea that women must “have courage still — and persevere,” regardless of the obstacles presented to her. Albright’s motto to “have courage still—and persevere” is the most prominent part of her speech, and remains consistent even when the rest of her speech shifts, which mirrors the outlook that Albright endorses.

Definition: appealing to credibility

Example:  “Letter from a Birmingham Jail” by Martin Luther King, Jr. (1963)

I think I should indicate why I am here in Birmingham since you have been influenced by the view which argues against "outsiders coming in." I have the honor of serving as president of the Southern Christian Leadership Conference, an organization operating in every southern state, with headquarters in Atlanta, Georgia. We have some eighty-five affiliated organizations across the South, and one of them is the Alabama Christian Movement for Human Rights. Frequently we share staff, educational and financial resources with our affiliates. Several months ago the affiliate here in Birmingham asked us to be on call to engage in a nonviolent direct action program if such were deemed necessary. We readily consented, and when the hour came we lived up to our promise. So I, along with several members of my staff, am here because I was invited here. I am here because I have organizational ties here.

Analysis:  King mentions that he is the “president of the Southern Christian Leadership Conference” that operates in “every southern state” and has “eighty-five affiliated organizations across the South.” He also emphasizes that he is in Birmingham because he was “invited” due to “organizational ties.” King spends a significant amount of time describing his credentials and his affiliation with the Church, which not only creates a common experience among the clergymen and himself but also establishes King as a respectable man with significant accomplishments. Because many white southerners believed that African Americans were inferior to themselves, King takes the time to appeal to his own credibility and authority in hopes that the clergymen will view him as their equal and will respect his message.

Note: please don’t write “appeals to ethos/pathos/logos.” Instead, try “appeals to credibility/emotion/logic,” or go further to describe specifically which emotion or credentials the author appeals to.

23. extended metaphor

Definition: differs from a regular metaphor in that several comparisons similar in theme are being made

Example:  “Is Google Making Us Stupid?” by Nicholas Carr (2008)

Over the past few years I’ve had an uncomfortable sense that someone, or something, has been tinkering with my brain, remapping the neural circuitry, reprogramming the memory. My mind isn’t going—so far as I can tell—but it’s changing. I’m not thinking the way I used to think. I can feel it most strongly when I’m reading. Immersing myself in a book or a lengthy article used to be easy. My mind would get caught up in the narrative or the turns of the argument, and I’d spend hours strolling through long stretches of prose.

Analysis:  Carr employs an extended metaphor  to liken his brain to a machine, suggesting that something “has been tinkering” with his brain, “remapping” and “reprogramming” his “neural circuitry.” By comparing his brain to a machine, Carr conveys his feeling that he is a slave to his computer and his sense of disconnectedness from his brain. Rather than being in harmony with his mind, he describes his brain as a separate entity. Carr’s metaphor also highlights the increasing influence of technology in modern life — so much so that our brains themselves have become computers.

24. imagery

Definition: descriptive language that provides vivid images that evoke the senses

Example:   Last Child in the Woods  by Richard Louv (2008)

In our useful boredom, we used our fingers to draw pictures on fogged glass as we watched telephone poles tick by. We saw birds on the wires and combines in the fields. We were fascinated with roadkill, and we counted cows and horses and coyotes and shaving-cream signs. We stared with a kind of reverence at the horizon, as thunderheads and dancing rain moved with us. We held our little plastic cars against the glass and pretended that they, too, were racing toward some unknown destination. We considered the past and dreamed of the future, and watched it all go by in the blink of an eye.

Analysis:  Louv recounts his experience staring out of the car window as a child with vivid imagery , describing watching “telephone poles tick by,” “birds on the wires,” “cows and horses and coyotes,” and “shaving-cream signs.” Louv jots seemingly disconnected images in short snippets, mimicking a car whizzing past an ever-changing landscape. The sharp images appeal to the reader’s sense of nostalgia as Louv allows them to witness their own youth “go by in the blink of an eye.”

25. inductive reasoning

Definition: reasoning that moves from specific observations to broader generalizations and theories; uses observations to detect patterns and regularities, and develops a hypothesis and later broader theories based on these observations (bottom-up approach)

Example:  “On Being a Cripple” by Nancy Mairs (1986)

"Cripple" seems to me a clean word, straightforward and precise. As a lover of words, I like the accuracy with which it describes my condition: I have lost the full use of my limbs. "Disabled," by contrast, suggests any incapacity, physical or mental. And I certainly don't like "handicapped," which implies that I have deliberately been put at a disadvantage, by whom I can't imagine (my God is not a Handicapper General), in order to equalize chances in the great race of life. These words seem to me to be moving away from my condition, to be widening the gap between word and reality. Most remote is the recently coined euphemism "differently-abled," which partakes of the same semantic hopefulness that transformed countries from "undeveloped" to "underdeveloped," then to "less developed," and finally to "developing" nations. People have continued to starve in those countries during the shift. Some realities do not obey the dictates of language.

Analysis:  Mairs begins by outlining her views on the word “cripple,” which “describes [her] condition” in a “straightforward and precise manner,” unlike vague terms such as “handicapped” and “differently-abled,” which widen “the gap between word and reality.” Much like “people have continued to starve” in underdeveloped nations despite the shift in nomenclature, Mairs scorns the “semantic hopefulness” that has led people to use less precise words to describe her condition, even though the disability itself cannot change. Mairs uses inductive reasoning  to conclude that “some realities do not obey the dictates of language” as she appeals to readers’ logic to deduce that using euphemisms to describe unfavorable circumstances is irrational and only serves to dilute the rectitude of precise language.

Definition: stating the opposite of what is said or meant

I hope you will treasure up the instructions which I have given you, and make them a guide to your feet and a light to your understanding. Build your character thoughtfully and painstakingly upon these precepts, and by and by, when you have got it built, you will be surprised and gratified to see how nicely and sharply it resembles everybody else’s.

Analysis:  Twain instructs youth to “treasure” his instructions and to construct their “character thoughtfully and painstakingly upon” the precepts they have read. However, Twain mentions that if they do so, they will be “surprised and gratified to see how nicely and sharply it resembles everybody else’s.” Twain’s irony  warns youth that if they simply obey their parents, they will not become a unique individual, and the unexpected ending to his satire reinforces his position that one should not mold themselves to meet societal norms.

27. juxtaposition

Definition: placing two or more things side by side for comparison or contrast

Example:   Silent Spring  by Rachel Carson (1962)

Along the roads, laurel, viburnum, and alder, great ferns and wildflowers delighted the traveler’s eye through much of the year. Even in winter, the roadsides were places of beauty, where countless birds came to feed on the berries and on the seed heads of the dried weeds rising above the snow. The countryside was, in fact, famous for the abundance and variety of its birdlife, and when the flood of migrants was pouring through in spring and fall people traveled from great distances to observe them. Others came to fish the streams, which flowed clear and cold out of the hills and contained shady pools where trout lay. So it had been from the days many years ago when the first settlers raised their houses, sank their wells, and built their barns.

Then a strange blight crept over the area and everything began to change. Some evil spell had settled on the community: mysterious maladies swept the flocks of chickens; the cattle and sheep sickened and died. Everywhere was a shadow of death. The farmers spoke of much illness among their families. In the town the doctors had become more and more puzzled by new kinds of sickness appearing among their patients. There had been several sudden and unexplained deaths, not only among adults but even among children, who would be stricken suddenly while at play and die within a few hours.

Analysis:  In her novel Silent Spring , Rachel Carson describes the beautiful American town with the cold, vapid town that it is destined to become due to climate change. She juxtaposes  the town’s “great ferns and wildflowers,” “birdlife,” and “clear and cold” streams with the “strange blight” that cast an “evil spell” on the community and the animals who have “sickened and died” from “mysterious maladies.” By creating such a sharp contrast between the present and the future, Carson coveys the magnitude of the climate crisis and emphasizes the urgency with which we must address it. Carson’s starkly contrasting images aim to evoke a strong emotional response in the reader that appeals to their sense of responsibility and citizenship.

Definition: appealing to logic

Example:  Greta Thunberg’s speech at the National Assembly in Paris (2019)

A lot of people, a lot of politicians, business leaders, journalists say they don't agree with what we are saying. They say we children are exaggerating, that we are alarmists. To answer this I would like to refer to page 108, chapter 2 in the latest IPCC report. There you will find all our "opinions" summarized because there you find a remaining carbon dioxide budget. Right there it says that if we are to have a sixty-seven percent chance of limiting the global temperature rise to below 1.5 degrees, we had on January 1st, 2018, 420 gigatons of carbon dioxide left in our CO2 budget. And of course, that number is much lower today. We emit about 42 gigatons of CO2 every year.

Analysis:  In her address to the National Assembly in Paris, Thunberg cites the 2018 “IPCC report” that outlines a total “remaining carbon dioxide budget” of “420 gigatons” in order to “have a sixty-seven percent chance of limiting the global temperature rise to below 1.5 degrees,” while “we emit about 42 gigatons of CO2 each year.” By citing specific data from a reputable scientific journal, Thunberg appeals to her audience’s logic; the data proves that the only viable option is to limit carbon dioxide emissions.

29. metonymy

Definition: a figure of speech consisting of the use of the name of one thing for that of another of which it is an attribute or with which it is associated

Example:  Margaret Thatcher’s eulogy for Ronald Reagan (2004)

Yet his ideas, so clear, were never simplistic. He saw the many sides of truth. Yes, he warned that the Soviet Union had an insatiable drive for military power and territorial expansion, yet he also sensed that it was being eaten away by systematic failures impossible to reform. Yes, he did not shrink from denouncing Moscow’s evil empire, but he realized that a man of goodwill might nonetheless emerge from its dark corridors.

Analysis:  In her eulogy for United States President Ronald Reagan, Thatcher refers to the Soviet Union as “Moscow’s evil empire.” Her metonymy  explicitly communicates a disdain for the Soviet Union, which establishes common ground between the United States and the United Kingdom, which helps Thatcher strengthen relations with the United States while eulogizing a friend.

Definition: the speed at which a piece of writing flows — use when discussing organization; point out where action/syntax begins to speed up, slow down, is interrupted, etc.

Example:   Notes on ‘Camp’  by Susan Sontag (1964)

1. To start very generally: Camp is a certain mode of aestheticism. It is one way of seeing the world as an aesthetic phenomenon. That way, the way of Camp, is not in terms of beauty, but in terms of the degree of artifice, of stylization.

2. To emphasize style is to slight content, or to introduce an attitude which is neutral with respect to content. It goes without saying that the Camp sensibility is disengaged, depoliticized — or at least apolitical.

Analysis:  Sontag writes Notes on ‘Camp’  as a “series of jottings” rather than in paragraph format in order to mimic the spontaneous and ever-changing nature of Camp. By presenting her notes as a numbered list, Sontag develops a quick, irregular pace  that is more fitting to describe the eccentricities of Camp. Because the notes are presented as a list, the ideas move by quickly, which further mirrors the whimsicality that is so characteristic of Camp.

31. paradox

Definition: apparently self-contradictory statement, the underlying meaning of which is revealed only by careful scrutiny; its purpose is to arrest attention and provoke fresh thought

Example:  “On the Writing of Essays” by Alexander Smith (1881)

He is the frankest, most outspoken of writers; and that very frankness and outspokenness puts the reader off his guard. If you wish to preserve your secret, wrap it up in frankness. The Essays are full of this trick. The frankness is as well simulated as the grape-branches of the Grecian artist which the birds flew towards and pecked. When Montaigne retreats, he does so like a skillful general, leaving his fires burning.

Analysis:  Smith describes Montaigne’s writing style as very frank and outspoken, asserting that “if you wish to preserve your secret, wrap it up in frankness.” Smith’s paradox , although outwardly nonsensical, forces the reader to pause and ruminate on the conflicting ideas, which naturally places emphasis on these ideas. Through his paradox, Smith suggests that an author’s works often contain intimate personal revelations that seem obvious, yet are often overlooked by most readers.

32. parallelism (parallel structure)

Definition: a repetition of sentences using the same grammatical structure emphasizing all aspects of the sentence equally

Example:  “Luckiest Man on the Face of the Earth” by Lou Gherig (1939)

When the New York Giants, a team you would give your right arm to beat, and vice versa, sends you a gift — that's something. When everybody down to the groundskeepers and those boys in white coats remembers you with trophies — that's something. When you have a wonderful mother-in-law who takes sides with you in squabbles with her own daughter — that's something. When you have a father and a mother who work all their lives so you can have an education and build your body — it's a blessing. When you have a wife who has been a tower of strength and shown more courage than you dreamed existed — that's the finest I know.

Analysis:  Gherig presents a series of parallel  sentences to emphasize his gratitude for the life he has lived. Because each sentence follows the same structure, Gherig’s list builds to a climax, which Gherig uses to enumerate his priorities and to emphasize his love for his family. Gherig further emphasizes his appreciation for his family even above his career by shifting from the phrase “that’s something” to describe his wife’s courage as “the finest” he knows. By breaking the pattern in his parallel sentences, Gherig focuses the attention on his family and loved ones, humbly placing his own successes on the back burner.

Definition: appealing to emotion

Example:  Viola Davis’s Women’s March Speech (2018)

I am speaking today not just for the 'Me Toos,' because I was a 'Me Too,' but when I raise my hand, I am aware of all the women who are still in silence. The women who are faceless. The women who don't have the money and don't have the constitution and who don't have the confidence and who don't have the images in our media that gives them a sense of self-worth enough to break their silence that is rooted in the shame of assault and rooted in the stigma of assault.

Analysis:  In her speech at the 2018 Women’s March, Viola Davis recognizes the millions of women who have been silently affected by sexual violence. She describes the women “don’t have the money,” “constitution,” or “confidence,” and those who still struggle with the “shame” and “stigma of assault.” Davis employs anaphora, repeating the phrase “don’t have” to evoke a sense of empathy for these women among the audience. By emphasizing that these victims “don’t have” the resources that many take for granted, Davis sheds light on the cruel reality that many victims still face due to the stigma surrounding sexual assault and women’s rights.

34. polysyndeton

Definition: the use of many conjunctions has the effect of slowing the pace or emphasizing the numerous words or clauses

Example:  “After the Storm” by Ernest Hemingway (1932)

I said, “Who killed him?” and he said, “I don’t know who killed him but he’s dead all right,” and it was dark and there was water standing in the street and no lights and windows broke and boats all up in the town and trees blown down and everything all blown and I got a skiff and went out and found my boat where I had her inside Mango Key and she was all right only she was full of water.

Analysis:  After learning of the murder, the narrator describes as “dark and there was water standing in the street and no lights and windows broke and boats all up in the town,” repeating the conjunction “and.” Hemingway employs

polysyndeton  to illustrate the narrator’s shock and panic following the murder. By inserting “and” between each phrase, Hemingway slows down the pace of the sentence, conveying the sense of the narrator’s surroundings moving in slow motion after hearing the news.

35. rhetorical question

Definition: a question presented by the author that is not meant to be answered

Example:  Clare de Booth Luce’s Speech at the Women’s National Press Club (1960)

For what is good journalism all about? On a working, finite level it is the effort to achieve illuminating candor in print and to strip away cant. It is the effort to do this not only in matters of state, diplomacy, and politics but also in every smaller aspect of life that touches the public interest or engages proper public curiosity.

Analysis:  In her speech at the Women’s National Press Club, de Booth asks the rhetorical question : “For what is good journalism all about?” in order to signal a shift in tone as she moves to describe the purpose of “good journalism.” By asking the audience a question, she invites them to consider their own motivations as journalists as she explains her own belief that “good journalism” is “the effort to achieve illuminating candor in print.” Rather than simply speaking about her views on journalism, de Booth expertly inserts a rhetorical question in order to evoke a moment of wonder and self-reflection in her audience before she answers her own question.

36. stream of consciousness

Definition: a technique that records the thoughts and feelings of a character without regard to logical argument or narrative sequence; reflects all the forces, internal and external, affecting the character's psyche at the moment

Example:  “Ain’t I a Woman” by Sojourner Truth (1851)

That man over there says that women need to be helped into carriages, and lifted over ditches, and to have the best place everywhere. Nobody ever helps me into carriages, or over mud-puddles, or gives me any best place! And ain't I a woman? Look at me! Look at my arm! I have ploughed and planted, and gathered into barns, and no man could head me! And ain't I a woman? I could work as much and eat as much as a man — when I could get it — and bear the lash as well! And ain't I a woman? I have borne thirteen children, and seen most all sold off to slavery, and when I cried out with my mother's grief, none but Jesus heard me! And ain't I a woman?

Analysis:  In 1851, Sojourner Truth gave a moving speech at the Women’s Rights Convention without preparation. Truth’s stream of consciousness  approach to the speech allows her to directly address her audience, beginning by mentioning “that man over there” and refuting his beliefs that women are fragile. Truth then moves to note that she has “ploughed and planted” more successfully than men, and she moves to the fact that she can “work as much and eat as much as a man.” She shifts yet again to recount that she has “borne thirteen children” and that “none but Jesus” heard her cry with her “mother’s grief” when they were sold to slavery. Albeit slightly messy, Truth’s lack of structure is effective because it reflects the never-ending struggles that African American women faced. When the injustices seemed to cease, another injustice would arise in a never-ending cycle of oppression. Truth’s speech thus resonated with many other women who had experienced the same struggles, and Truth became a powerful voice in the fight racial and gender equality.

37. synecdoche

Definition: the rhetorical substitution of a part for the whole

Example:  “Falling Down is Part of Growing Up” by Henry Petroski (1985)

We are transported across impromptu bridges of arms thrown up without plans or blueprints between mother and aunt, between neighbor and father, between brother and sister — none of whom is a registered structural engineer. We come to Mama and to Papa eventually to forget our scare reflex and we learn to trust the beams and girders and columns of their arms and our cribs.

Analysis:  Petroski refers to a child’s parents and crib as “beams and girders and columns” that the child must trust, emphasizing the structural aspect of a young child’s support system. Instead of referring to the parents and crib as a whole, Petroski uses synecdoche  to strip away the sentimental connotations associated with a mother’s arms and a baby’s crib, highlighting only the “beams and girders and columns” that prevent the child from falling and returning to his novel’s central topic of engineering.

Definition: the structure of sentences and/or phrases

We have waited for more than 340 years for our constitutional and God-given rights. The nations of Asia and Africa are moving with jetlike speed toward gaining political independence, but we still creep at horse and buggy pace toward gaining a cup of coffee at a lunch counter. Perhaps it is easy for those who have never felt the stinging darts of segregation to say, "Wait." But when you have seen vicious mobs lynch your mothers and fathers at will and drown your sisters and brothers at whim; when you have seen hate-filled policemen curse, kick and even kill your black brothers and sisters; when you see the vast majority of your twenty million Negro brothers smothering in an airtight cage of poverty in the midst of an affluent society; when you suddenly find your tongue twisted and your speech stammering as you seek to explain to your six-year-old daughter why she can't go to the public amusement park that has just been advertised on television, and see tears welling up in her eyes when she is told that Funtown is closed to colored children, and see ominous clouds of inferiority beginning to form in her little mental sky, and see her beginning to distort her personality by developing an unconscious bitterness toward white people; when you have to concoct an answer for a five-year-old son who is asking: "Daddy, why do white people treat colored people so mean?"; when you take a cross-country drive and find it necessary to sleep night after night in the uncomfortable corners of your automobile because no motel will accept you; when you are humiliated day in and day out by nagging signs reading "white" and "colored"; when your first name becomes "nigger," your middle name becomes "boy" (however old you are) and your last name becomes "John," and your wife and mother are never given the respected title "Mrs."; when you are harried by day and haunted by night by the fact that you are a Negro, living constantly at tiptoe stance, never quite knowing what to expect next, and are plagued with inner fears and outer resentments; when you are forever fighting a degenerating sense of "nobodiness" — then you will understand why we find it difficult to wait.

Analysis:  In his “Letter from a Birmingham Jail,” King addresses those who instruct him to “wait” for racial equality by describing the “stinging pain of segregation” as seeing “vicious mobs lynch your mothers and fathers,” seeing “hate-filled policemen curse, kick, and even kill your brothers and sisters,” and seeing the “tears welling up” in your six-year-old daughter’s eyes when “she is told that Funtown is closed to colored children,” among a host of other horrific images. Rather than using several shorter sentences to describe segregation, King uses a single sentence, separated by numerous semicolons. King’s choice of syntax  mirrors the never-ending reach of segregation and racial inequality. While the sentence consists of a string of short images, it pauses on a longer phrase in which King describes finding his “tongue-twisted” as he explains to his “six-year-old daughter why she can't go to the public amusement park that has just been advertised on television,” and seeing “tears welling up in her eyes when she is told that Funtown is closed to colored children” while he watches the “ominous clouds of inferiority beginning to form in her little mental sky” and her “distort her personality by developing an unconscious bitterness toward white people.” By making this phrase significantly longer than his other images, King allows the reader to pause and ruminate on the idea of a young girl losing her innocence to an unjust world. King appeals to the reader’s emotions as he conveys such a heartbreaking image.

Definition: a statement of purpose, intent, or main idea in a literary work

Example:   Notes on ‘Camp’  by Susan Sontag

58. The ultimate Camp statement: it's good because  it's awful . . . Of course, one can't always say that. Only under certain conditions, those which I've tried to sketch in these notes.

Analysis:  Sontag places her thesis  at the end of her Notes on ‘Camp’ , which allows her to summarize her list and to assert that Camp is “good because  it’s awful.” Sontag concludes the notes by referencing her sporadic list of musings regarding Camp as a whole and declaring them the “conditions” under which Camp can be both good and awful.

the use of stylistic devices that reveal an author’s attitude towards a subject

Example:  “If Black English Isn’t a Language, Then Tell Me, What Is?” by James Baldwin (1979)

I say that the present skirmish is rooted in American history, and it is. Black English is the creation of the black diaspora. Blacks came to the United States chained to each other, but from different tribes: Neither could speak the other's language. If two black people, at that bitter hour of the world's history, had been able to speak to each other, the institution of chattel slavery could never have lasted as long as it did. Subsequently, the slave was given, under the eye, and the gun, of his master, Congo Square, and the Bible–or in other words, and under these conditions, the slave began the formation of the black church, and it is within this unprecedented tabernacle that black English began to be formed. This was not, merely, as in the European example, the adoption of a foreign tongue, but an alchemy that transformed ancient elements into a new language: A language comes into existence by means of brutal necessity, and the rules of the language are dictated by what the language must convey.

Analysis:  Baldwin adopts a formal, academic tone , assessing the development of “Black English” through a historical lens. Baldwin concludes that “Black English is the creation of the black diaspora” as “an alchemy that transformed ancient elements into a new language.” By using academic diction, Baldwin approaches the development of Black English not as a cultural or social issue, but simply as a historical phenomenon that should be studied objectively, which allows him to persuade his readers that Black English should be considered a distinct language.

👉 Play Kahoot with AP Lang teacher Kathryn Howard as she recaps rhetorical strategies and devices!  

One last disclaimer: Fiveable is an educational company without political or religious affiliations and it neither endorses nor opposes any views expressed in the above passages. There you go! When looking at each device and its corresponding example, think of ways and reasons authors integrate these rhetorical devices, styles, and terms into their writing! Thinking that much ahead will pay off when you write the Rhetorical Analysis essay in May! 😄

Fiveable

Stay Connected

© 2024 Fiveable Inc. All rights reserved.

AP® and SAT® are trademarks registered by the College Board, which is not affiliated with, and does not endorse this website.

What are your chances of acceptance?

Calculate for all schools, your chance of acceptance.

Duke University

Your chancing factors

Extracurriculars.

examples of ap lang rhetorical essays

How to Write the AP Lang Argument Essay + Examples

What’s covered:, what is the ap language argument essay, tips for writing the ap language argument essay, ap english language argument essay examples, how will ap scores impact my college chances.

In 2023, over 550,148 students across the U.S. took the AP English Language and Composition Exam, and 65.2% scored higher than a 3. The AP English Language Exam tests your ability to analyze a piece of writing, synthesize information, write a rhetorical essay, and create a cohesive argument. In this post, we’ll be discussing the best way to approach the argumentative essay section of the test, and we’ll give you tips and tricks so you can write a great essay.

The AP English Language Exam as of 2023 is structured as follows:

Section 1: 45 multiple choice questions to be completed in an hour. This portion counts for 45% of your score. This section requires students to analyze a piece of literature. The questions ask about its content and/or what could be edited within the passage.

Section 2: Three free response questions to be completed in the remaining two hours and 15 minutes. This section counts for 55% of your score. These essay questions include the synthesis essay, the rhetorical essay, and the argumentative essay.

  • Synthesis essay: Read 6-7 sources and create an argument using at least three of the sources.
  • Rhetorical analysis essay: Describe how a piece of writing evokes meaning and symbolism.
  • Argumentative essay: Pick a side of a debate and create an argument based on evidence. In this essay, you should develop a logical argument in support of or against the given statement and provide ample evidence that supports your conclusion. Typically, a five paragraph format is great for this type of writing. This essay is scored holistically from 1 to 9 points.

Do you want more information on the structure of the full exam? Take a look at our in-depth overview of the AP Language and Composition Exam .

Although the AP Language Argument may seem daunting at first, once you understand how the essay should be structured, it will be a lot easier to create cohesive arguments.

Below are some tips to help you as you write the essay.

1. Organize your essay before writing

Instead of jumping right into your essay, plan out what you will say beforehand. It’s easiest to make a list of your arguments and write out what facts or evidence you will use to support each argument. In your outline, you can determine the best order for your arguments, especially if they build on each other or are chronological. Having a well-organized essay is crucial for success.

2. Pick one side of the argument, but acknowledge the other side

When you write the essay, it’s best if you pick one side of the debate and stick with it for the entire essay. All your evidence should be in support of that one side. However, in your introductory paragraph, as you introduce the debate, be sure to mention any merit the arguments of the other side has. This can make the essay a bit more nuanced and show that you did consider both sides before determining which one was better. Often, acknowledging another viewpoint then refuting it can make your essay stronger.

3. Provide evidence to support your claims

The AP readers will be looking for examples and evidence to support your argument. This doesn’t mean that you need to memorize a bunch of random facts before the exam. This just means that you should be able to provide concrete examples in support of your argument.

For example, if the essay topic is about whether the role of the media in society has been detrimental or not, and you argue that it has been, you may talk about the phenomenon of “fake news” during the 2016 presidential election.

AP readers are not looking for perfect examples, but they are looking to see if you can provide enough evidence to back your claim and make it easily understood.

4. Create a strong thesis statement

The thesis statement will set up your entire essay, so it’s important that it is focused and specific, and that it allows for the reader to understand your body paragraphs. Make sure your thesis statement is the very last sentence of your introductory paragraph. In this sentence, list out the key points you will be making in the essay in the same order that you will be writing them. Each new point you mention in your thesis should start a paragraph in your essay.

Below is a prompt and sample student essay from the May 2019 exam . We’ll look at what the student did well in their writing and where they could improve.

Prompt: “The term “overrated” is often used to diminish concepts, places, roles, etc. that the speaker believes do not deserve the prestige they commonly enjoy; for example, many writers have argued that success is overrated, a character in a novel by Anthony Burgess famously describes Rome as a “vastly overrated city,” and Queen Rania of Jordan herself has asserted that “[b]eing queen is overrated.”

Select a concept, place, role, etc. to which you believe that the term “overrated” should be applied. Then, write a well-developed essay in which you explain your judgment. Use appropriate evidence from your reading, experience, or observations to support your argument.

Sample Student Essay #1:

[1] Competition is “overrated.” The notion of motivation between peers has evolved into a source of unnecessary stress and even lack of morals. Whether it be in an academic environment or in the industry, this new idea of competition is harmful to those competing and those around them.

[2] Back in elementary school, competition was rather friendly. It could have been who could do the most pushups or who could get the most imaginary points in a classroom for a prize. If you couldn’t do the most pushups or win that smelly sticker, you would go home and improve yourself – there would be no strong feelings towards anyone, you would just focus on making yourself a better version of yourself. Then as high school rolled around, suddenly applying for college doesn’t seem so far away –GPA seems to be that one stat that defines you – extracurriculars seem to shape you – test scores seem to categorize you. Sleepless nights, studying for the next day’s exam, seem to become more and more frequent. Floating duck syndrome seems to surround you (FDS is where a competitive student pretends to not work hard but is furiously studying beneath the surface just like how a duck furiously kicks to stay afloat). All of your competitors appear to hope you fail – but in the end what do you and your competitor’s gain? Getting one extra point on the test? Does that self-satisfaction compensate for the tremendous amounts of acquired stress? This new type of “competition” is overrated – it serves nothing except a never-ending source of anxiety and seeks to weaken friendships and solidarity as a whole in the school setting.

[3] A similar idea of “competition” can be applied to business. On the most fundamental level, competition serves to be a beneficial regulator of prices and business models for both the business themselves and consumers. However, as businesses grew increasingly greedy and desperate, companies resorted to immoral tactics that only hurt their reputations and consumers as a whole. Whether it be McDonald’s coupons that force you to buy more food or tech companies like Apple intentionally slowing down your iPhone after 3 years to force you to upgrade to the newest device, consumers suffer and in turn speak down upon these companies. Similar to the evolved form of competition in school, this overrated form causes pain for all parties and has since diverged from the encouraging nature that the principle of competition was “founded” on.

The AP score for this essay was a 4/6, meaning that it captured the main purpose of the essay but there were still substantial parts missing. In this essay, the writer did a good job organizing the sections and making sure that their writing was in order according to the thesis statement. The essay first discusses how competition is harmful in elementary school and then discusses this topic in the context of business. This follows the chronological order of somebody’s life and flows nicely.

The arguments in this essay are problematic, as they do not provide enough examples of how exactly competition is overrated. The essay discusses the context in which competition is overrated but does not go far enough in explaining how this connects to the prompt.

In the first example, school stress is used to explain how competition manifests. This is a good starting point, but it does not talk about why competition is overrated; it simply mentions that competition can be unhealthy. The last sentence of that paragraph is the main point of the argument and should be expanded to discuss how the anxiety of school is overrated later on in life. 

In the second example, the writer discusses how competition can lead to harmful business practices, but again, this doesn’t reflect the reason this would be overrated. Is competition really overrated because Apple and McDonald’s force you to buy new products? This example could’ve been taken one step farther. Instead of explaining why business structures—such as monopolies—harm competition, the author should discuss how those particular structures are overrated.

Additionally, the examples the writer used lack detail. A stronger essay would’ve provided more in-depth examples. This essay seemed to mention examples only in passing without using them to defend the argument.

It should also be noted that the structure of the essay is incomplete. The introduction only has a thesis statement and no additional context. Also, there is no conclusion paragraph that sums up the essay. These missing components result in a 4/6.

Now let’s go through the prompt for a sample essay from the May 2022 exam . The prompt is as follows:

Colin Powell, a four-star general and former United States Secretary of State, wrote in his 1995 autobiography: “[W]e do not have the luxury of collecting information indefinitely. At some point, before we can have every possible fact in hand, we have to decide. The key is not to make quick decisions, but to make timely decisions.”

Write an essay that argues your position on the extent to which Powell’s claim about making decisions is valid. 

In your response you should do the following:

  • Respond to the prompt with a thesis that presents a defensible position. 
  • Provide evidence to support your line of reasoning. 
  • Explain how the evidence supports your line of reasoning. 
  • Use appropriate grammar and punctuation in communicating your argument.

Sample Student Essay #2:

Colin Powell, who was a four star general and a former United States Secretary of State. He wrote an autobiography and had made a claim about making decisions. In my personal opinion, Powell’s claim is true to full extent and shows an extremely valuable piece of advice that we do not consider when we make decisions.

Powell stated, “before we can have every possible fact in hand we have to decide…. but to make it a timely decision” (1995). With this statement Powell is telling the audience of his autobiography that it does not necessarily matter how many facts you have, and how many things you know. Being able to have access to everything possible takes a great amount of time and we don’t always have all of the time in the world. A decision has to be made with what you know, waiting for something else to come in while trying to make a decision whether that other fact is good or bad you already have a good amount of things that you know. Everyone’s time is valuable, including yours. At the end of the day the decision will have to be made and that is why it should be made in a “timely” manner.

This response was graded for a score of 2/6. Let’s break down the score to smaller points that signify where the student fell short.

The thesis in this essay is clearly outlined at the end of the first paragraph. The student states their agreement with Powell’s claim and frames the rest of their essay around this stance. The success in scoring here lies in the clear communication of the thesis and the direction the argument will take. It’s important to make the thesis statement concise, specific, and arguable, which the student has successfully done.

While the student did attempt to provide evidence to support their thesis, it’s clear that their explanation lacks specific detail and substance. They referenced Powell’s statement, but did not delve into how this statement has proven true in specific instances, and did not provide examples that could bring the argument to life.

Commentary is an essential part of this section’s score. It means explaining the significance of the evidence and connecting it back to the thesis. Unfortunately, the student’s commentary here is too vague and does not effectively elaborate on how the evidence supports their argument.

To improve, the student could use more concrete examples to demonstrate their point and discuss how each piece of evidence supports their thesis. For instance, they could discuss specific moments in Powell’s career where making a timely decision was more valuable than waiting for all possible facts. This would help illustrate the argument in a more engaging, understandable way.

A high score in the “sophistication” category of the grading rubric is given for demonstrating a complex understanding of the rhetorical situation (purpose, audience, context, etc.), making effective rhetorical choices, or establishing a line of reasoning. Here, the student’s response lacks complexity and sophistication. They’ve simply agreed with Powell’s claim and made a few general statements without providing a deeper analysis or effectively considering the rhetorical situation.

To increase sophistication, the student could explore possible counterarguments or complexities within Powell’s claim. They could discuss potential drawbacks of making decisions without all possible facts, or examine situations where timely decisions might not yield the best results. By acknowledging and refuting these potential counterarguments, they could add more depth to their analysis and showcase their understanding of the complexities involved in decision-making.

The student could also analyze why Powell, given his background and experiences, might have come to such a conclusion, thus providing more context and showing an understanding of the rhetorical situation.

Remember, sophistication in argumentation isn’t about using fancy words or complicated sentences. It’s about showing that you understand the complexity of the issue at hand and that you’re able to make thoughtful, nuanced arguments. Sophistication shows that you can think critically about the topic and make connections that aren’t immediately obvious.

Now that you’ve looked at an example essay and some tips for the argumentative essay, you know how to better prepare for the AP English Language and Composition Exam.

While your AP scores don’t usually impact your admissions chances , colleges do care a lot about your course rigor. So, taking as many APs as you can will certainly boost your chances! AP scores can be a way for high-performing students to set themselves apart, particularly when applying to prestigious universities. Through the process of self-reporting scores , you can show your hard work and intelligence to admissions counselors.

That said, the main benefit of scoring high on AP exams comes once you land at your dream school, as high scores can allow you to “test out” of entry-level requirements, often called GE requirements or distribution requirements. This will save you time and money.

To understand how your course rigor stacks up, check out CollegeVine’s free chancing engine . This resource takes your course rigor, test scores, extracurriculars, and more, to determine your chances of getting into over 1600 colleges across the country!

Related CollegeVine Blog Posts

examples of ap lang rhetorical essays

Calculate for all schools

Your chance of acceptance, your chancing factors, extracurriculars, ap lang rhetorical analysis essay examples.

Hi everyone! I'm looking for some really good examples of rhetorical analysis essays for AP Lang. Would love to see how others have approached this type of essay and learn from them!

Hello! I'm glad you're looking for examples to learn and improve your rhetorical analysis skills. To guide you, I'll point out a few resources and tips that could be helpful.

1. College Board resources: The College Board, which oversees the AP program, provides sample essays and scoring guidelines on their website. You can find past exam questions and sample responses for AP Lang, including rhetorical analysis essays, in their "Free-Response Questions" section. These examples include the grading commentary that explains what makes the essays effective. Here's the link: https://apcentral.collegeboard.org/courses/ap-english-language-and-composition/exam/past-exam-questions

2. Analyzing other AP Lang prompts: It's also useful to look at the rhetorical analysis prompts from past exams and try to analyze them yourself. Practice identifying the rhetorical strategies used and make notes on what you think works well and what could be improved. This will help you understand the process of writing an effective rhetorical analysis.

3. Check the AP Lang subreddit or online forums: The AP Lang subreddit and other online forums can be a great place to find examples of rhetorical analysis essays written by other students. Be sure to read the comments, as they often contain valuable feedback and insight from peers and experienced individuals. Remember, though, to take these examples with a grain of salt, as they may not be as high quality or well vetted as official College Board examples.

4. Get feedback: As you practice writing your own rhetorical analysis essays, try to get feedback from your teacher, classmates, or an experienced individual who is familiar with AP Lang expectations. This will help you refine your writing and understand what areas you need to improve.

When examining samples, focus on understanding how the writer:

- Clearly states their thesis, identifying the rhetorical strategies they'll analyze

- Organizes their essay, often using a chronological or point-by-point structure

- Analyzes specific examples from the text and explains their significance

- Uses effective transitions between points and examples

- Builds a coherent and well-supported argument throughout the essay

By studying examples and practicing your own writing, you'll be well equipped to tackle the AP Lang rhetorical analysis essay. Good luck!

About CollegeVine’s Expert FAQ

CollegeVine’s Q&A seeks to offer informed perspectives on commonly asked admissions questions. Every answer is refined and validated by our team of admissions experts to ensure it resonates with trusted knowledge in the field.

PrepScholar

Choose Your Test

Sat / act prep online guides and tips, how to write a perfect synthesis essay for the ap language exam.

author image

Advanced Placement (AP)

body-pencil-sharpen-notebook-1

If you're planning to take the AP Language (or AP Lang) exam , you might already know that 55% of your overall exam score will be based on three essays. The first of the three essays you'll have to write on the AP Language exam is called the "synthesis essay." If you want to earn full points on this portion of the AP Lang Exam, you need to know what a synthesis essay is and what skills are assessed by the AP Lang synthesis essay.

In this article, we'll explain the different aspects of the AP Lang synthesis essay, including what skills you need to demonstrate in your synthesis essay response in order to achieve a good score. We'll also give you a full breakdown of a real AP Lang Synthesis Essay prompt, provide an analysis of an AP Lang synthesis essay example, and give you four tips for how to write a synthesis essay.

Let's get started by taking a closer look at how the AP Lang synthesis essay works!

Synthesis Essay AP Lang: What It Is and How It Works

The AP Lang synthesis essay is the first of three essays included in the Free Response section of the AP Lang exam.

The AP Lang synthesis essay portion of the Free Response section lasts for one hour total . This hour consists of a recommended 15 minute reading period and a 40 minute writing period. Keep in mind that these time allotments are merely recommendations, and that exam takers can parse out the allotted 60 minutes to complete the synthesis essay however they choose.

Now, here's what the structure of the AP Lang synthesis essay looks like. The exam presents six to seven sources that are organized around a specific topic (like alternative energy or eminent domain, which are both past synthesis exam topics).

Of these six to seven sources, at least two are visual , including at least one quantitative source (like a graph or pie chart, for example). The remaining four to five sources are print text-based, and each one contains approximately 500 words.

In addition to six to seven sources, the AP Lang exam provides a written prompt that consists of three paragraphs. The prompt will briefly explain the essay topic, then present a claim that students will respond to in an essay that synthesizes material from at least three of the sources provided.

Here's an example prompt provided by the College Board:

Directions : The following prompt is based on the accompanying six sources.

This question requires you to integrate a variety of sources into a coherent, well-written essay. Refer to the sources to support your position; avoid mere paraphrase or summary. Your argument should be central; the sources should support this argument .

Remember to attribute both direct and indirect citations.

Introduction

Television has been influential in United States presidential elections since the 1960's. But just what is this influence, and how has it affected who is elected? Has it made elections fairer and more accessible, or has it moved candidates from pursuing issues to pursuing image?

Read the following sources (including any introductory information) carefully. Then, in an essay that synthesizes at least three of the sources for support, take a position that defends, challenges, or qualifies the claim that television has had a positive impact on presidential elections.

Refer to the sources as Source A, Source B, etc.; titles are included for your convenience.

Source A (Campbell) Source B (Hart and Triece) Source C (Menand) Source D (Chart) Source E (Ranney) Source F (Koppel)

Like we mentioned earlier, this prompt gives you a topic — which it briefly explains — then asks you to take a position. In this case, you'll have to choose a stance on whether television has positively or negatively affected U.S. elections. You're also given six sources to evaluate and use in your response. Now that you have everything you need, now your job is to write an amazing synthesis essay.

But what does "synthesize" mean, exactly? According to the CollegeBoard, when an essay prompt asks you to synthesize, it means that you should "combine different perspectives from sources to form a support of a coherent position" in writing. In other words, a synthesis essay asks you to state your claim on a topic, then highlight the relationships between several sources that support your claim on that topic. Additionally, you'll need to cite specific evidence from your sources to prove your point.

The synthesis essay counts for six of the total points on the AP Lang exam . Students can receive 0-1 points for writing a thesis statement in the essay, 0-4 based on incorporation of evidence and commentary, and 0-1 points based on sophistication of thought and demonstrated complex understanding of the topic.

You'll be evaluated based on how effectively you do the following in your AP Lang synthesis essay:

Write a thesis that responds to the exam prompt with a defensible position

Provide specific evidence that to support all claims in your line of reasoning from at least three of the sources provided, and clearly and consistently explain how the evidence you include supports your line of reasoning

Demonstrate sophistication of thought by either crafting a thoughtful argument, situating the argument in a broader context, explaining the limitations of an argument

Make rhetorical choices that strengthen your argument and/or employ a vivid and persuasive style throughout your essay.

If your synthesis essay meets the criteria above, then there's a good chance you'll score well on this portion of the AP Lang exam!

If you're looking for even more information on scoring, the College Board has posted the AP Lang Free Response grading rubric on its website. ( You can find it here. ) We recommend taking a close look at it since it includes additional details about the synthesis essay scoring.

body-chisel-break-apart

Don't be intimidated...we're going to teach you how to break down even the hardest AP synthesis essay prompt.

Full Breakdown of a Real AP Lang Synthesis Essay Prompt

In this section, we'll teach you how to analyze and respond to a synthesis essay prompt in five easy steps, including suggested time frames for each step of the process.

Step 1: Analyze the Prompt

The very first thing to do when the clock starts running is read and analyze the prompt. To demonstrate how to do this, we'll look at the sample AP Lang synthesis essay prompt below. This prompt comes straight from the 2018 AP Lang exam:

Eminent domain is the power governments have to acquire property from private owners for public use. The rationale behind eminent domain is that governments have greater legal authority over lands within their dominion than do private owners. Eminent domain has been instituted in one way or another throughout the world for hundreds of years.

Carefully read the following six sources, including the introductory information for each source. Then synthesize material from at least three of the sources and incorporate it into a coherent, well-developed essay that defends, challenges, or qualifies the notion that eminent domain is productive and beneficial.

Your argument should be the focus of your essay. Use the sources to develop your argument and explain the reasoning for it. Avoid merely summarizing the sources. Indicate clearly which sources you are drawing from, whether through direct quotation, paraphrase, or summary. You may cite the sources as Source A, Source B, etc., or by using the descriptions in parentheses.

On first read, you might be nervous about how to answer this prompt...especially if you don't know what eminent domain is! But if you break the prompt down into chunks, you'll be able to figure out what the prompt is asking you to do in no time flat.

To get a full understanding of what this prompt wants you to do, you need to identify the most important details in this prompt, paragraph by paragraph. Here's what each paragraph is asking you to do:

  • Paragraph 1: The prompt presents and briefly explains the topic that you'll be writing your synthesis essay about. That topic is the concept of eminent domain.
  • Paragraph 2: The prompt presents a specific claim about the concept of eminent domain in this paragraph: Eminent domain is productive and beneficial. This paragraph instructs you to decide whether you want to defend, challenge, or qualify that claim in your synthesis essay , and use material from at least three of the sources provided in order to do so.
  • Paragraph 3: In the last paragraph of the prompt, the exam gives you clear instructions about how to approach writing your synthesis essay . First, make your argument the focus of the essay. Second, use material from at least three of the sources to develop and explain your argument. Third, provide commentary on the material you include, and provide proper citations when you incorporate quotations, paraphrases, or summaries from the sources provided.

So basically, you'll have to agree with, disagree with, or qualify the claim stated in the prompt, then use at least three sources substantiate your answer. Since you probably don't know much about eminent domain, you'll probably decide on your position after you read the provided sources.

To make good use of your time on the exam, you should spend around 2 minutes reading the prompt and making note of what it's asking you to do. That will leave you plenty of time to read the sources provided, which is the next step to writing a synthesis essay.

Step 2: Read the Sources Carefully

After you closely read the prompt and make note of the most important details, you need to read all of the sources provided. It's tempting to skip one or two sources to save time--but we recommend you don't do this. That's because you'll need a thorough understanding of the topic before you can accurately address the prompt!

For the sample exam prompt included above, there are six sources provided. We're not going to include all of the sources in this article, but you can view the six sources from this question on the 2018 AP Lang exam here . The sources include five print-text sources and one visual source, which is a cartoon.

As you read the sources, it's important to read quickly and carefully. Don't rush! Keep your pencil in hand to quickly mark important passages that you might want to use as evidence in your synthesis. While you're reading the sources and marking passages, you want to think about how the information you're reading influences your stance on the issue (in this case, eminent domain).

When you finish reading, take a few seconds to summarize, in a phrase or sentence, whether the source defends, challenges, or qualifies whether eminent domain is beneficial (which is the claim in the prompt) . Though it might not feel like you have time for this, it's important to give yourself these notes about each source so you know how you can use each one as evidence in your essay.

Here's what we mean: say you want to challenge the idea that eminent domain is useful. If you've jotted down notes about each source and what it's saying, it will be easier for you to pull the relevant information into your outline and your essay.

So how much time should you spend reading the provided sources? The AP Lang exam recommends taking 15 minutes to read the sources . If you spend around two of those minutes reading and breaking down the essay prompt, it makes sense to spend the remaining 13 minutes reading and annotating the sources.

If you finish reading and annotating early, you can always move on to drafting your synthesis essay. But make sure you're taking your time and reading carefully! It's better to use a little extra time reading and understanding the sources now so that you don't have to go back and re-read the sources later.

body-weightlifting-lift-strong

A strong thesis will do a lot of heavy lifting in your essay. (See what we did there?)

Step 3: Write a Strong Thesis Statement

After you've analyzed the prompt and thoroughly read the sources, the next thing you need to do in order to write a good synthesis essay is write a strong thesis statement .

The great news about writing a thesis statement for this synthesis essay is that you have all the tools you need to do it at your fingertips. All you have to do in order to write your thesis statement is decide what your stance is in relationship to the topic provided.

In the example prompt provided earlier, you're essentially given three choices for how to frame your thesis statement: you can either defend, challenge, or qualify a claim that's been provided by the prompt, that eminent domain is productive and beneficial . Here's what that means for each option:

If you choose to defend the claim, your job will be to prove that the claim is correct . In this case, you'll have to show that eminent domain is a good thing.

If you choose to challenge the claim, you'll argue that the claim is incorrect. In other words, you'll argue that eminent domain isn't productive or beneficial.

If you choose to qualify, that means you'll agree with part of the claim, but disagree with another part of the claim. For instance, you may argue that eminent domain can be a productive tool for governments, but it's not beneficial for property owners. Or maybe you argue that eminent domain is useful in certain circumstances, but not in others.

When you decide whether you want your synthesis essay to defend, challenge, or qualify that claim, you need to convey that stance clearly in your thesis statement. You want to avoid simply restating the claim provided in the prompt, summarizing the issue without making a coherent claim, or writing a thesis that doesn't respond to the prompt.

Here's an example of a thesis statement that received full points on the eminent domain synthesis essay:

Although eminent domain can be misused to benefit private interests at the expense of citizens, it is a vital tool of any government that intends to have any influence on the land it governs beyond that of written law.

This thesis statement received full points because it states a defensible position and establishes a line of reasoning on the issue of eminent domain. It states the author's position (that some parts of eminent domain are good, but others are bad), then goes on to explain why the author thinks that (it's good because it allows the government to do its job, but it's bad because the government can misuse its power.)

Because this example thesis statement states a defensible position and establishes a line of reasoning, it can be elaborated upon in the body of the essay through sub-claims, supporting evidence, and commentary. And a solid argument is key to getting a six on your synthesis essay for AP Lang!

Looking for help studying for your AP exam? Our one-on-one online AP tutoring services can help you prepare for your AP exams. Get matched with a top tutor who got a high score on the exam you're studying for!

Step 4: Create a Bare-Bones Essay Outline

Once you've got your thesis statement drafted, you have the foundation you need to develop a bare bones outline for your synthesis essay. Developing an outline might seem like it's a waste of your precious time, but if you develop your outline well, it will actually save you time when you start writing your essay.

With that in mind, we recommend spending 5 to 10 minutes outlining your synthesis essay . If you use a bare-bones outline like the one below, labeling each piece of content that you need to include in your essay draft, you should be able to develop out the most important pieces of the synthesis before you even draft the actual essay.

To help you see how this can work on test day, we've created a sample outline for you. You can even memorize this outline to help you out on test day! In the outline below, you'll find places to fill in a thesis statement, body paragraph topic sentences, evidence from the sources provided, and commentary :

  • Present the context surrounding the essay topic in a couple of sentences (this is a good place to use what you learned about the major opinions or controversies about the topic from reading your sources).
  • Write a straightforward, clear, and concise thesis statement that presents your stance on the topic
  • Topic sentence presenting first supporting point or claim
  • Evidence #1
  • Commentary on Evidence #1
  • Evidence #2 (if needed)
  • Commentary on Evidence #2 (if needed)
  • Topic sentence presenting second supporting point or claim
  • Topic sentence presenting three supporting point or claim
  • Sums up the main line of reasoning that you developed and defended throughout the essay
  • Reiterates the thesis statement

Taking the time to develop these crucial pieces of the synthesis in a bare-bones outline will give you a map for your final essay. Once you have a map, writing the essay will be much easier.

Step 5: Draft Your Essay Response

The great thing about taking a few minutes to develop an outline is that you can develop it out into your essay draft. After you take about 5 to 10 minutes to outline your synthesis essay, you can use the remaining 30 to 35 minutes to draft your essay and review it.

Since you'll outline your essay before you start drafting, writing the essay should be pretty straightforward. You'll already know how many paragraphs you're going to write, what the topic of each paragraph will be, and what quotations, paraphrases, or summaries you're going to include in each paragraph from the sources provided. You'll just have to fill in one of the most important parts of your synthesis—your commentary.

Commentaries are your explanation of why your evidence supports the argument you've outlined in your thesis. Your commentary is where you actually make your argument, which is why it's such a critical part of your synthesis essay.

When thinking about what to say in your commentary, remember one thing the AP Lang synthesis essay prompt specifies: don't just summarize the sources. Instead, as you provide commentary on the evidence you incorporate, you need to explain how that evidence supports or undermines your thesis statement . You should include commentary that offers a thoughtful or novel perspective on the evidence from your sources to develop your argument.

One very important thing to remember as you draft out your essay is to cite your sources. The AP Lang exam synthesis essay prompt indicates that you can use generic labels for the sources provided (e.g. "Source 1," "Source 2," "Source 3," etc.). The exam prompt will indicate which label corresponds with which source, so you'll need to make sure you pay attention and cite sources accurately. You can cite your sources in the sentence where you introduce a quote, summary, or paraphrase, or you can use a parenthetical citation. Citing your sources affects your score on the synthesis essay, so remembering to do this is important.

body-green-arrow-down

Keep reading for a real-life example of a great AP synthesis essay response!

Real-Life AP Synthesis Essay Example and Analysis

If you're still wondering how to write a synthesis essay, examples of real essays from past AP Lang exams can make things clearer. These real-life student AP synthesis essay responses can be great for helping you understand how to write a synthesis essay that will knock the graders' socks off .

While there are multiple essay examples online, we've chosen one to take a closer look at. We're going to give you a brief analysis of one of these example student synthesis essays from the 2019 AP Lang Exam below!

Example Synthesis Essay AP Lang Response

To get started, let's look at the official prompt for the 2019 synthesis essay:

In response to our society's increasing demand for energy, large-scale wind power has drawn attention from governments and consumers as a potential alternative to traditional materials that fuel our power grids, such as coal, oil, natural gas, water, or even newer sources such as nuclear or solar power. Yet the establishment of large-scale, commercial-grade wind farms is often the subject of controversy for a variety of reasons.

Carefully read the six sources, found on the AP English Language and Composition 2019 Exam (Question 1), including the introductory information for each source. Write an essay that synthesizes material from at least three of the sources and develops your position on the most important factors that an individual or agency should consider when deciding whether to establish a wind farm.

Source A (photo) Source B (Layton) Source C (Seltenrich) Source D (Brown) Source E (Rule) Source F (Molla)

In your response you should do the following:

  • Respond to the prompt with a thesis presents a defensible position.
  • Select and use evidence from at least 3 of the provided sources to support your line of reasoning. Indicate clearly the sources used through direct quotation, paraphrase, or summary. Sources may be cited as Source A, Source B, etc., or by using the description in parentheses.
  • Explain how the evidence supports your line of reasoning.
  • Use appropriate grammar and punctuation in communicating your argument.

Now that you know exactly what the prompt asked students to do on the 2019 AP Lang synthesis essay, here's an AP Lang synthesis essay example, written by a real student on the AP Lang exam in 2019:

[1] The situation has been known for years, and still very little is being done: alternative power is the only way to reliably power the changing world. The draw of power coming from industry and private life is overwhelming current sources of non-renewable power, and with dwindling supplies of fossil fuels, it is merely a matter of time before coal and gas fuel plants are no longer in operation. So one viable alternative is wind power. But as with all things, there are pros and cons. The main factors for power companies to consider when building wind farms are environmental boon, aesthetic, and economic factors.

[2] The environmental benefits of using wind power are well-known and proven. Wind power is, as qualified by Source B, undeniably clean and renewable. From their production requiring very little in the way of dangerous materials to their lack of fuel, besides that which occurs naturally, wind power is by far one of the least environmentally impactful sources of power available. In addition, wind power by way of gearbox and advanced blade materials, has the highest percentage of energy retention. According to Source F, wind power retains 1,164% of the energy put into the system – meaning that it increases the energy converted from fuel (wind) to electricity 10 times! No other method of electricity production is even half that efficient. The efficiency and clean nature of wind power are important to consider, especially because they contribute back to power companies economically.

[3] Economically, wind power is both a boon and a bone to electric companies and other users. For consumers, wind power is very cheap, leading to lower bills than from any other source. Consumers also get an indirect reimbursement by way of taxes (Source D). In one Texan town, McCamey, tax revenue increased 30% from a wind farm being erected in the town. This helps to finance improvements to the town. But, there is no doubt that wind power is also hurting the power companies. Although, as renewable power goes, wind is incredibly cheap, it is still significantly more expensive than fossil fuels. So, while it is helping to cut down on emissions, it costs electric companies more than traditional fossil fuel plants. While the general economic trend is positive, there are some setbacks which must be overcome before wind power can take over as truly more effective than fossil fuels.

[4] Aesthetics may be the greatest setback for power companies. Although there may be significant economic and environmental benefit to wind power, people will always fight to preserve pure, unspoiled land. Unfortunately, not much can be done to improve the visual aesthetics of the turbines. White paint is the most common choice because it "[is] associated with cleanliness." (Source E). But, this can make it stand out like a sore thumb, and make the gargantuan machines seem more out of place. The site can also not be altered because it affects generating capacity. Sound is almost worse of a concern because it interrupts personal productivity by interrupting people's sleep patterns. One thing for power companies to consider is working with turbine manufacturing to make the machines less aesthetically impactful, so as to garner greater public support.

[5] As with most things, wind power has no easy answer. It is the responsibility of the companies building them to weigh the benefits and the consequences. But, by balancing economics, efficiency, and aesthetics, power companies can create a solution which balances human impact with environmental preservation.

And that's an entire AP Lang synthesis essay example, written in response to a real AP Lang exam prompt! It's important to remember AP Lang exam synthesis essay prompts are always similarly structured and worded, and students often respond in around the same number of paragraphs as what you see in the example essay response above.

Next, let's analyze this example essay and talk about what it does effectively, where it could be improved upon, and what score past exam scorers awarded it.

To get started on an analysis of the sample synthesis essay, let's look at the scoring commentary provided by the College Board:

  • For development of thesis, the essay received 1 out of 1 possible points
  • For evidence and commentary, the essay received 4 out of 4 possible points
  • For sophistication of thought, the essay received 0 out of 1 possible points.

This means that the final score for this example essay was a 5 out of 6 possible points . Let's look more closely at the content of the example essay to figure out why it received this score breakdown.

Thesis Development

The thesis statement is one of the three main categories that is taken into consideration when you're awarded points on this portion of the exam. This sample essay received 1 out of 1 total points.

Now, here's why: the thesis statement clearly and concisely conveys a position on the topic presented in the prompt--alternative energy and wind power--and defines the most important factors that power companies should consider when deciding whether to establish a wind farm.

Evidence and Commentary

The second key category taken into consideration when synthesis exams are evaluated is incorporation of evidence and commentary. This sample received 4 out of 4 possible points for this portion of the synthesis essay. At bare minimum, this sample essay meets the requirement mentioned in the prompt that the writer incorporate evidence from at least three of the sources provided.

On top of that, the writer does a good job of connecting the incorporated evidence back to the claim made in the thesis statement through effective commentary. The commentary in this sample essay is effective because it goes beyond just summarizing what the provided sources say. Instead, it explains and analyzes the evidence presented in the selected sources and connects them back to supporting points the writer makes in each body paragraph.

Finally, the writer of the essay also received points for evidence and commentary because the writer developed and supported a consistent line of reasoning throughout the essay . This line of reasoning is summed up in the fourth paragraph in the following sentence: "One thing for power companies to consider is working with turbine manufacturing to make the machines less aesthetically impactful, so as to garner greater public support."

Because the writer did a good job consistently developing their argument and incorporating evidence, they received full marks in this category. So far, so good!

Sophistication of Thought

Now, we know that this essay received a score of 5 out of 6 total points, and the place where the writer lost a point was on the basis of sophistication of thought, for which the writer received 0 out of 1 points. That's because this sample essay makes several generalizations and vague claims where it could have instead made specific claims that support a more balanced argument.

For example, in the following sentence from the 5th paragraph of the sample essay, the writer misses the opportunity to state specific possibilities that power companies should consider for wind energy . Instead, the writer is ambiguous and non-committal, saying, "As with most things, wind power has no easy answer. It is the responsibility of the companies building them to weigh the benefits and consequences."

If the writer of this essay was interested in trying to get that 6th point on the synthesis essay response, they could consider making more specific claims. For instance, they could state the specific benefits and consequences power companies should consider when deciding whether to establish a wind farm. These could include things like environmental impacts, economic impacts, or even population density!

Despite losing one point in the last category, this example synthesis essay is a strong one. It's well-developed, thoughtfully written, and advances an argument on the exam topic using evidence and support throughout.

body-number-four-post-it-note

4 Tips for How to Write a Synthesis Essay

AP Lang is a timed exam, so you have to pick and choose what you want to focus on in the limited time you're given to write the synthesis essay. Keep reading to get our expert advice on what you should focus on during your exam.

Tip 1: Read the Prompt First

It may sound obvious, but when you're pressed for time, it's easy to get flustered. Just remember: when it comes time to write the synthesis essay, read the prompt first !

Why is it so important to read the prompt before you read the sources? Because when you're aware of what kind of question you're trying to answer, you'll be able to read the sources more strategically. The prompt will help give you a sense of what claims, points, facts, or opinions to be looking for as you read the sources.

Reading the sources without having read the prompt first is kind of like trying to drive while wearing a blindfold: you can probably do it, but it's likely not going to end well!

Tip 2: Make Notes While You Read

During the 15-minute reading period at the beginning of the synthesis essay, you'll be reading through the sources as quickly as you can. After all, you're probably anxious to start writing!

While it's definitely important to make good use of your time, it's also important to read closely enough that you understand your sources. Careful reading will allow you to identify parts of the sources that will help you support your thesis statement in your essay, too.

As you read the sources, consider marking helpful passages with a star or check mark in the margins of the exam so you know which parts of the text to quickly re-read as you form your synthesis essay. You might also consider summing up the key points or position of each source in a sentence or a few words when you finish reading each source during the reading period. Doing so will help you know where each source stands on the topic given and help you pick the three (or more!) that will bolster your synthesis argument.

Tip 3: Start With the Thesis Statement

If you don't start your synthesis essay with a strong thesis statement, it's going to be tough to write an effective synthesis essay. As soon as you finish reading and annotating the provided sources, the thing you want to do next is write a strong thesis statement.

According to the CollegeBoard grading guidelines for the AP Lang synthesis essay, a strong thesis statement will respond to the prompt— not restate or rephrase the prompt. A good thesis will take a clear, defensible position on the topic presented in the prompt and the sources.

In other words, to write a solid thesis statement to guide the rest of your synthesis essay, you need to think about your position on the topic at hand and then make a claim about the topic based on your position. This position will either be defending, challenging, or qualifying the claim made in the essay's prompt.

The defensible position that you establish in your thesis statement will guide your argument in the rest of the essay, so it's important to do this first. Once you have a strong thesis statement, you can begin outlining your essay.

Tip 4: Focus on Your Commentary

Writing thoughtful, original commentary that explains your argument and your sources is important. In fact, doing this well will earn you four points (out of a total of six)!

AP Lang provides six to seven sources for you on the exam, and you'll be expected to incorporate quotations, paraphrases, or summaries from at least three of those sources into your synthesis essay and interpret that evidence for the reader.

While incorporating evidence is very important, in order to get the extra point for "sophistication of thought" on the synthesis essay, it's important to spend more time thinking about your commentary on the evidence you choose to incorporate. The commentary is your chance to show original thinking, strong rhetorical skills, and clearly explain how the evidence you've included supports the stance you laid out in your thesis statement.

To earn the 6th possible point on the synthesis essay, make sure your commentary demonstrates a nuanced understanding of the source material, explains this nuanced understanding, and places the evidence incorporated from the sources in conversation with each other. To do this, make sure you're avoiding vague language. Be specific when you can, and always tie your commentary back to your thesis!

body-person-arrows-next

What's Next?

There's a lot more to the AP Language exam than just the synthesis essay. Be sure to check out our expert guide to the entire exam , then learn more about the tricky multiple choice section .

Is the AP Lang exam hard...or is it easy? See how it stacks up to other AP tests on our list of the hardest AP exams .

Did you know there are technically two English AP exams? You can learn more about the second English AP test, the AP Literature exam, in this article . And if you're confused about whether you should take the AP Lang or AP Lit test , we can help you make that decision, too.

Want to improve your SAT score by 160 points or your ACT score by 4 points?   We've written a guide for each test about the top 5 strategies you must be using to have a shot at improving your score. Download them for free now:

Ashley Sufflé Robinson has a Ph.D. in 19th Century English Literature. As a content writer for PrepScholar, Ashley is passionate about giving college-bound students the in-depth information they need to get into the school of their dreams.

Ask a Question Below

Have any questions about this article or other topics? Ask below and we'll reply!

Improve With Our Famous Guides

  • For All Students

The 5 Strategies You Must Be Using to Improve 160+ SAT Points

How to Get a Perfect 1600, by a Perfect Scorer

Series: How to Get 800 on Each SAT Section:

Score 800 on SAT Math

Score 800 on SAT Reading

Score 800 on SAT Writing

Series: How to Get to 600 on Each SAT Section:

Score 600 on SAT Math

Score 600 on SAT Reading

Score 600 on SAT Writing

Free Complete Official SAT Practice Tests

What SAT Target Score Should You Be Aiming For?

15 Strategies to Improve Your SAT Essay

The 5 Strategies You Must Be Using to Improve 4+ ACT Points

How to Get a Perfect 36 ACT, by a Perfect Scorer

Series: How to Get 36 on Each ACT Section:

36 on ACT English

36 on ACT Math

36 on ACT Reading

36 on ACT Science

Series: How to Get to 24 on Each ACT Section:

24 on ACT English

24 on ACT Math

24 on ACT Reading

24 on ACT Science

What ACT target score should you be aiming for?

ACT Vocabulary You Must Know

ACT Writing: 15 Tips to Raise Your Essay Score

How to Get Into Harvard and the Ivy League

How to Get a Perfect 4.0 GPA

How to Write an Amazing College Essay

What Exactly Are Colleges Looking For?

Is the ACT easier than the SAT? A Comprehensive Guide

Should you retake your SAT or ACT?

When should you take the SAT or ACT?

Stay Informed

examples of ap lang rhetorical essays

Get the latest articles and test prep tips!

Looking for Graduate School Test Prep?

Check out our top-rated graduate blogs here:

GRE Online Prep Blog

GMAT Online Prep Blog

TOEFL Online Prep Blog

Holly R. "I am absolutely overjoyed and cannot thank you enough for helping me!”

Purdue Online Writing Lab Purdue OWL® College of Liberal Arts

Welcome to the Purdue Online Writing Lab

OWL logo

Welcome to the Purdue OWL

This page is brought to you by the OWL at Purdue University. When printing this page, you must include the entire legal notice.

Copyright ©1995-2018 by The Writing Lab & The OWL at Purdue and Purdue University. All rights reserved. This material may not be published, reproduced, broadcast, rewritten, or redistributed without permission. Use of this site constitutes acceptance of our terms and conditions of fair use.

The Online Writing Lab at Purdue University houses writing resources and instructional material, and we provide these as a free service of the Writing Lab at Purdue. Students, members of the community, and users worldwide will find information to assist with many writing projects. Teachers and trainers may use this material for in-class and out-of-class instruction.

The Purdue On-Campus Writing Lab and Purdue Online Writing Lab assist clients in their development as writers—no matter what their skill level—with on-campus consultations, online participation, and community engagement. The Purdue Writing Lab serves the Purdue, West Lafayette, campus and coordinates with local literacy initiatives. The Purdue OWL offers global support through online reference materials and services.

A Message From the Assistant Director of Content Development 

The Purdue OWL® is committed to supporting  students, instructors, and writers by offering a wide range of resources that are developed and revised with them in mind. To do this, the OWL team is always exploring possibilties for a better design, allowing accessibility and user experience to guide our process. As the OWL undergoes some changes, we welcome your feedback and suggestions by email at any time.

Please don't hesitate to contact us via our contact page  if you have any questions or comments.

All the best,

Social Media

Facebook twitter.

IMAGES

  1. Ap Lang Rhetorical Analysis Essay Example

    examples of ap lang rhetorical essays

  2. AP Language and Composition argumentative essay

    examples of ap lang rhetorical essays

  3. AP Language-- Rhetorical devices A-D

    examples of ap lang rhetorical essays

  4. 60+ Rhetorical Devices with Examples for Effective Persuasion • 7ESL

    examples of ap lang rhetorical essays

  5. ⭐ Ap english language and composition rhetorical analysis essay sample

    examples of ap lang rhetorical essays

  6. How To Study For Ap Lang Exam

    examples of ap lang rhetorical essays

VIDEO

  1. AP Lang

  2. Providing Meaningful Feedback on Student Writing

  3. How Can I Master the Art of Writing a Rhetorical Analysis Essay for AP Lang Q2?

  4. Writing Rhetorical Analysis was HARD Until I Did THIS!

  5. Top 10 Rhetorical Devices You Need to Know

  6. AP Lang Rhetorical Analysis Rubric

COMMENTS

  1. How to Write the AP Lang Rhetorical Analysis Essay (With Example)

    AP Lang Rhetorical Analysis Essay Example (Continued) Toward the end of the speech, Obama states that change happens "not mainly through the exploits of the famous and the powerful, but through the countless acts of often anonymous courage and kindness" (lines 78-81). Through carefully chosen diction that portrays her as a quiet, regular ...

  2. AP English Language and Composition Past Exam Questions

    Download free-response questions from past exams along with scoring guidelines, sample responses from exam takers, and scoring distributions. If you are using assistive technology and need help accessing these PDFs in another format, contact Services for Students with Disabilities at 212-713-8333 or by email at [email protected]. Note ...

  3. How to Write the AP Lang Rhetorical Essay

    AP Lang Rhetorical Analysis Essay Example Below is a prompt and example for a rhetorical essay, along with its score and what the writer did well and could have improved: The passage below is an excerpt from "On the Want of Money," an essay written by nineteenth-century author William Hazlitt. Read the passage carefully.

  4. AP Lang

    Timing. You have 40 minutes to complete the rhetorical analysis essay for AP Lang: 12 minutes: Read the text and plan out your essay. (TOBI) 6 minutes: Write your introduction paragraph. 18 minutes: Write 2-3 body paragraphs. 2 minutes: Write a quick conclusion. 2 minutes: Proofread and revise your essay.

  5. PDF Sample Student Responses

    AP English Language and Composition Rhetorical Analysis Free-Response Question (2020) Sample Student Responses . 3 . Sample B [1] On April 9, 1964, United States First Lady Claudia Johnson gave a speech in honor of Eleanor Roosevelt. She spoke to those in attendance of the luncheon, specifically talking to the women within the crowd.

  6. AP Lang

    AP Lang Rhetorical Analysis Essay Practice. Rhetorical Analysis practice is one of the most important ways to prepare for the exam! Review student writing practice samples and corresponding feedback from TA Brandon Wu! While you don't need to memorize every rhetorical device for the exam, you should take some time to familiarize yourself with them.

  7. PDF AP English Language and Composition Question 2: Rhetorical Analysis

    AP English Language and Composition Question 2: Rhetorical Analysis (2019) Sample Student Responses 6 Sample HH [1] In the 1930, Indian were oppressed by the British. Britain had a monopoly on the taxation of salt. This unjust tax and control led to the rise of Mohandas "Mahatma" Gandhi as the leader of the Salt March.

  8. 3 AP® English Language Rhetorical Essay Strategies

    The AP® English Language rhetorical essay can be nightmare inducing for some AP® students, but there is no need for fear. In this exam review we will lay out helpful strategies to get you through the rhetorical essays in no time. Rhetorical Strategy #1: Dissecting the Prompt. The first rhetorical essay strategy is to dissect the prompt.

  9. AP English Language and Composition: Sample Rhetorical Analysis and

    AP English Language and Composition: Sample Argument Question. The following paragraph is adapted from Mirror for Man, a book written by anthropologist Clyde Kluckhorn in the middle of the twentieth century. Read the passage carefully. Then, write an essay that examines the extent to which the author's characterization of the United States ...

  10. How to Ace the AP Language Rhetorical Analysis Essay

    In this video, I'll show you how to write the AP English Language rhetorical analysis essay (Q2) step by step using the actual 2017 prompt. Watch me annotate...

  11. AP Lang

    This is a list of main rhetorical device terms that you should know for the exam as well as definitions & examples for each. These terms will mostly show up on the multiple-choice section, so it's important to be able to identify them in a work of writing, but you won't actually have to use the device in your own writing.

  12. PDF Sample Student Responses

    AP English Language and Composition Rhetorical Analysis Free-Response Question (2020) Sample Student Responses 1 Sample A [1] Many Americans admired Kennedy and his administration when he was in the White House. He was a loved man. Many share in fond memories of Kennedy up until his death, when the nation grieved for him.

  13. PDF ap06 english lang student samples

    adequately. Essays earning a score of 6 respond to the prompt . They adequately analyze the rhetorical strategies Hazlitt uses to develop his position about money. These essays may refer to the passage explicitly or implicitly. The writing may contain lapses in diction or syntax, but generally the prose is clear.

  14. PDF AP English Language and Composition

    1 − Essays earning a score of 1 meet the criteria for the score of 2 but are undeveloped, especially simplistic in their explanation, or weak in their control of language. 0 - Indicates an off-topic response, one that merely repeats the prompt, an entirely crossed-out response, a drawing, or a response in a language other than English.

  15. Expert Guide to the AP Language and Composition Exam

    The AP English Language and Composition Multiple-Choice. The multiple-choice section tests you on two main areas. The first is how well you can read and understand nonfiction passages for their use of rhetorical devices and tools. The second is how well you can "think like a writer" and make revisions to texts in composition questions.

  16. How to Write the AP Lang Argument Essay + Examples

    2. Pick one side of the argument, but acknowledge the other side. When you write the essay, it's best if you pick one side of the debate and stick with it for the entire essay. All your evidence should be in support of that one side. However, in your introductory paragraph, as you introduce the debate, be sure to mention any merit the ...

  17. PDF AP English Language and Composition

    examples test takers used. The Question Leader continues, "Students connected to the topic in surprising, powerful, and beautiful ways. That level of connection extended to even the lower-scoring essays. Even responses that struggled with the abstraction of 'perfection,' could still articulate a position." Students were

  18. PDF AP® ENGLISH LANGUAGE AND COMPOSITION

    Sample: 2A Score: 8. This essay effectively identifies and analyzes three of Cesar Chavez's rhetorical choices — striking diction, juxtaposition, and appeals to reader's fundamental moral beliefs — to argue that "nonviolence is the best and most moral way to bring change.". Providing convincing evidence and analysis (for example ...

  19. AP Lang rhetorical analysis essay examples?

    This will help you understand the process of writing an effective rhetorical analysis. 3. Check the AP Lang subreddit or online forums: The AP Lang subreddit and other online forums can be a great place to find examples of rhetorical analysis essays written by other students.

  20. How to Write a Perfect Synthesis Essay for the AP Language Exam

    While there are multiple essay examples online, we've chosen one to take a closer look at. We're going to give you a brief analysis of one of these example student synthesis essays from the 2019 AP Lang Exam below! Example Synthesis Essay AP Lang Response. To get started, let's look at the official prompt for the 2019 synthesis essay:

  21. Rhetorical analysis essay tips (how I got a 5!)

    318K subscribers in the APStudents community. No matter what course you are taking, we are a community that helps students earn college credit!

  22. PDF AP English Language and Composition

    AP® English Language and Composition 2022 Scoring Guidelines. Argument Essay 6 points . Colin Powell, a four-star general and former United States secretary of state, wrote in his 1995 autobiography: "[W]e do not have the luxury of collecting information indefinitely. At some point, before we can have every possible fact in hand, we have to ...

  23. Welcome to the Purdue Online Writing Lab

    Mission. The Purdue On-Campus Writing Lab and Purdue Online Writing Lab assist clients in their development as writers—no matter what their skill level—with on-campus consultations, online participation, and community engagement. The Purdue Writing Lab serves the Purdue, West Lafayette, campus and coordinates with local literacy initiatives.

  24. AP English Language and Composition

    Section II: Free Response. 3 Questions | 2 hours 15 minutes (includes a 15-minute reading period | 55% of Exam Score. Students write essays that respond to 3 free-response prompts from the following categories: Synthesis Question: After reading 6-7 texts about a topic (including visual and quantitative sources), students will compose an ...